SQE Flashcards

1
Q

A man has instructed a solicitor to act for him on the purchase of a house. The man intends to finance his purchase with a mortgage. The solicitor has just received the mortgage offer, which contains the following condition:

‘The solicitor must ensure that existing credit card debt of £3,500 is paid off prior to drawdown of the loan.’

The solicitor has reported fully to the client on the terms of the offer. After completion, the mortgage lender discovers that the man still owes £3,500 on the credit card debt referred to in the mortgage offer.

Did the solicitor breach any duties with respect to the man’s existing £3,500 credit card debt?

A No, because the solicitor has reported to the clients on the terms of the offer, which is the extent of the solicitor’s obligation to the lender.No, because the solicitor has reported to the clients on the terms of the offer, which is the extent of the solicitor’s obligation to the lender.

B No, because the mortgage offer is between the borrower and the lender, and it is not the solicitor’s concern if the borrower fails to comply with a condition in the offer.No, because the mortgage offer is between the borrower and the lender, and it is not the solicitor’s concern if the borrower fails to comply with a condition in the offer.

C Yes, because the UK Finance Mortgage Lender’s Handbook requires solicitors to ensure their client’s existing debts are fully paid before submitting the certificate of title to the lender.Yes, because the UK Finance Mortgage Lender’s Handbook requires solicitors to ensure their client’s existing debts are fully paid before submitting the certificate of title to the lender.

D Yes, because the condition contained in the mortgage offer has not been complied with.Yes, because the condition contained in the mortgage offer has not been complied with.

E Yes, because if the borrower defaults on the loan, the solicitor can be pursued for any shortfall incurred by the lender.

A

Yes, because the condition contained in the mortgage offer has not been complied with.

The solicitor breached the condition in the mortgage offer. The condition in the offer required the instructing solicitor to ensure that the existing credit card debt was paid off; thus, the failure to do so is a breach of the solicitor’s duty to the lender.

How well did you know this?
1
Not at all
2
3
4
5
Perfectly
2
Q

After a competitor pushed an opponent while playing hockey, the opponent became angry and hit the competitor with his stick, drawing a penalty. The opponent was not ejected because the penalty was not far outside the bounds of normal play. The competitor had to be taken to the hospital for stitches. Wanting to make an example of an athlete who plays sports too aggressively, local authorities charged the opponent with inflicting grievous bodily harm under Offences Against the Person Act 1861 section 20.

Which of the following is the opponent’s best defence?

A Self-defence.

B Diminished responsibility.

C Loss of control.

D Lack of intent.Lack of intent.

E Consent.

A

(E) The competitor’s best defence is consent.

Consent can be used as a defence against a charge of inflicting grievous bodily harm. In contact sports such as rugby and hockey, consent will be implied so long as the actions are not greatly outside the rules of the game. Since the competitor here was not ejected, we can assume his actions were within the bounds of consent.

How well did you know this?
1
Not at all
2
3
4
5
Perfectly
3
Q

A woman died last month, having never made a will. The woman is survived by her 20-year-old daughter and her 19-year-old son, who the woman adopted when he was a baby. The woman’s parents and brother are also alive. All members of the family are keen to administer the woman’s estate.

Who has the best right to apply for a grant of representation to the woman’s estate?

A The woman’s daughter only.

B The woman’s parents.

C The woman’s two children.

D The woman’s son.

E The woman’s brother only.

A

(C) The woman’s children have the best right to apply for a grant of representation. This is an intestacy situation, and the woman’s estate will be administered under a grant of letters of administration.

The order of entitlement to a grant of letters of administration is set out in rule 22 of the Non-Contentious Probate Rules (‘NCPR’), under which the women’s children have the best right to act as administrators. No distinction is made between natural and adopted children in this respect.

How well did you know this?
1
Not at all
2
3
4
5
Perfectly
4
Q

A woman died on 17 May 2023. At the date of her death, she owned the following assets:

  • A house worth £390,000
  • Chattels and cash with a value of £70,000
  • Shares held in an individual savings account (ISA) worth £60,000

At the date of her death, the woman owed income tax of £25,000 in respect of the tax year 2022/23.

The woman left £100,000 of her estate to a registered charity and left the remainder of her estate to her daughter.

What is the woman’s chargeable estate for inheritance tax purposes?

A £335,000

B £395,000

C £420,000

D £520,000

E £265,000

A

(B) £395,000

To calculate the chargeable estate, we add up all the assets the woman owned at death that are not exempt and subtract out her liabilities and gifts made to charities. None of the woman’s assets are exempt, so her gross estate is £520,000 (£390,000 house + £70,000 chattels and cash + £60,000 ISA). From that, we subtract the income tax owed (£520,000 - £25,000 = £495,000) and the gift to the charity (£495,000 - £100,000 = £395,000)

How well did you know this?
1
Not at all
2
3
4
5
Perfectly
5
Q

A woman, who is illiterate, makes a will. On the day the will is to be executed at a solicitor’s office, the woman tells her friend that she is unable to travel to the office that day but that the friend should execute the will without her. The friend signs the will for the woman at the solicitor’s office, which had been planned all along due to the fact that the woman cannot read or write. In addition, the friend acts as one of two witnesses to the execution of the will.

Which of the following best describes the legal position regarding the validity of the will?

A The will is invalid as it was not signed by the woman herself.

B The will is invalid as the friend who signed it also acted as a witness.

C The will is invalid as the woman was not present when the will was signed.

D The will is invalid as it was not appropriately witnessed.

E The will is valid.

A

The will is invalid as the woman was not present when the will was signed.

The will is invalid as the woman was not present when the will was signed. To be valid, a will must be in writing and signed by or on behalf of the testator in the presence of two or more witnesses present at the same time who each sign the will in the presence of the testator. It is possible for another person to sign on behalf of the testator if they are blind or illiterate. However, the testator must be present, and the will should be read to them before the will is signed on their behalf. Here, the woman was not present when the will was signed, so the will is invalid.

How well did you know this?
1
Not at all
2
3
4
5
Perfectly
6
Q

A man and his friend are arguing. As the argument intensifies, the man and the friend each start pushing each other. The friend falls over and hits his head, cracking his skull. The man did not intend for his friend to fall over or crack his skull. The friend is taken to hospital. In the ambulance, the friend dies of a heart attack completely unrelated to the fall.

Which of the following is the most appropriate charge that the prosecution could bring against the man?

A Murder.

B Voluntary manslaughter.

C Involuntary manslaughter.

D Grievous bodily harm with intent under section 18.

E Grievous bodily harm under section 20.

A

(E) Section 20 grievous bodily harm is the most appropriate charge. It applies when the defendant causes a wound or serious bodily harm, intending to cause or being reckless about causing some harm. Here, the pushing was unlawful as it caused the friend to crack his skull.

How well did you know this?
1
Not at all
2
3
4
5
Perfectly
7
Q

In her will, a woman left the residue of her estate to trustees on trust to invest and use the income to pay grants to the children of employees and former employees of her family company to enable them to attend university.

Which of the following statements best describes the reason why this is not a valid charitable trust?

A It is not for the public benefit.

B The purpose is not of a charitable nature.

C The terms of the trust contravene the rule against inalienability.

D The objects are not sufficiently certain.

E The objects are not exclusively charitable.

A

(A) The trust is not a valid charitable trust because it is not for the public benefit. A charitable trust must be for a charitable purpose as defined in the Charities Act 2011, it must be for the public benefit, and its objects must be exclusively charitable. Here, the people who can benefit from the trust are defined by reference to a private relationship – the employment of their parents by the family company. This means they do not form an adequate cross-section of the public, and so the trust fails the public benefit test.

How well did you know this?
1
Not at all
2
3
4
5
Perfectly
8
Q

A husband discovers his wife is having an affair. He is so furious he cannot contain himself and decides to kill her. The husband gets the shotgun and ammunition he keeps in the shed. He drinks a bottle of whiskey to get up the courage to go ahead with it. He lies in wait for his wife at their marital home. After two hours, the wife returns home. The husband shoots her once in the back. The wife sustains serious injury but eventually recovers. The husband is charged with attempted murder. The husband is subsequently diagnosed with severe depression.

What defence, if any, is available to the husband?

A Loss of control.

B Diminished responsibility.

C Self-defence.

D Intoxication.

E None of the above.

A

(E) None of these defences is available to the husband.

Loss of control and diminished responsibility are both partial defences to murder, reducing the conviction to voluntary manslaughter. They are not available to attempted murder, and so (A) and (B) are incorrect. (C) is incorrect because self-defence is available when reasonable force is used to defend oneself, another, or property, which clearly does not apply here. Intoxication can offer a defence to a specific intent offence like attempted murder if the defendant is so intoxicated that they cannot form the necessary mens rea. Here, we are told that the husband intended to kill his wife, so this defence does not apply

How well did you know this?
1
Not at all
2
3
4
5
Perfectly
9
Q

In breach of trust, a trustee removed £10,000 from the trust’s bank account and deposited it into his personal bank current account. He then received £10,000 as a lottery win and paid that into his account. The trustee then removed £10,000 from his account and used it to pay his credit card debt.

Can the trust beneficiaries recover the £10,000 remaining in the trustee’s account?

A No, because the trustee mixed the trust funds with his own money.

B No, because of the rule that the first money into the account is the first money out of the account.

C No, because where two funds are mixed in the same account, they are shared proportionately.

D Yes, because the trustee is deemed to have spent his own funds first.

E Yes, because the trustee did not earn the lottery winnings but won them through luck.

A

D. Yes, because the trustee is deemed to have spent his own funds first.

(D) The beneficiaries can recover the £10,000 remaining in the trustee’s personal account because the trustee is deemed to have spent his own funds first. When a trustee places trust funds into an account with the trustee’s own money, the beneficiaries may claim a charge over the account for the amount of trust funds in it. If the trustee has drawn money out of the account, the general rule is that the trustee is treated as withdrawing their own money first. Therefore, the trustee is deemed to have spent his own £10,000 on paying his credit card debt, and the beneficiaries can claim the remaining £10,000 in the account. (A) is, therefore, incorrect. (B) is incorrect because the first-in, first-out rule applies when a trustee has mixed the funds of two trusts in a personal current account. (C) is incorrect because the proportionate solution applies when the court displaces the first-in, first-out rule for mixed trust funds. (E) is incorrect because the fact that the trustee won the lottery money does not affect the beneficiaries’ ability to claim it

How well did you know this?
1
Not at all
2
3
4
5
Perfectly
10
Q

A solicitor is representing a defendant at court for first appearance. The defendant is charged with theft. The evidence against the defendant is overwhelmingly strong, and the solicitor believes the defendant should plead guilty.

Should the solicitor tell the defendant to plead guilty?

A No, because the decision on plea is for the defendant only and a solicitor must never tell their client how to plead.

B No, because the solicitor should tell the defendant to plead not guilty and elect Crown Court trial, as acquittal rates are higher in the Crown Court.

C Yes, because the sentence will be lower than if he is convicted after trial.

D Yes, because the evidence against the defendant is overwhelmingly strong.

E Yes, because it is the solicitor’s professional opinion.

A

No, because the decision on plea is for the defendant only and a solicitor must never tell their client how to plead.

(A) The solicitor should not tell the defendant to plead guilty. The decision on plea is for the defendant only, and a solicitor must never tell their client how to plead. A defence solicitor should advise on the strength of the evidence, which includes warning the defendant when the evidence is strong. A solicitor should also advise on the credit defendants are entitled to for an early guilty plea compared to conviction following trial. However, the decision on plea remains the defendant’s alone. (B) and (E) are incorrect as a solicitor should never tell their client how to plead. (B) is incorrect because the solicitor should not tell the defendant to plead guilty or not guilty. (C), (D), and (E) are incorrect, because, although all three statements are true, the solicitor should not tell the defendant how to plead.

How well did you know this?
1
Not at all
2
3
4
5
Perfectly
11
Q

A woman makes a will in which she gives the residue of her estate to a named friend “to be distributed as I have instructed him”. Before the will was signed, the woman sent a letter to her friend in which she listed her intended beneficiaries. The woman dies. The friend has not responded to the letter.

Which of the following best describes the position of the friend following the woman’s death?

A The friend holds the money on resulting trust for the estate.

B The gift to the friend fails, and the estate passes on the woman’s intestacy.

C The friend may keep the money because he did not accept the trust.

D The friend holds the money on trust to distribute as directed in the letter.

E The friend may keep the money because the beneficiaries were not named in the will.

A

The friend holds the money on trust to distribute as directed in the letter.

(D) The friend holds the money on trust to distribute as directed in the letter. The usual rule is that where a trust is to take effect on death, all its terms must be included in a valid will. However, where the will declares a trust but does not identify the beneficiaries, a valid half-secret trust arises provided that the terms of the trust were communicated to the trustee before the will was made and the communication is consistent with the wording of the will. The secret trustee must accept the trust either expressly or impliedly by silence. Here, the will refers to a communication which was made before the date of the will and is consistent with the letter already sent. Therefore the half-secret trust is valid, and the friend must distribute the money as directed in the letter. (A) and (B) are incorrect because the half-secret trust is valid. (C) and (E) are incorrect because it is clear on the face of the will that the friend is a trustee, and so he may in no circumstances keep the money for himself.

How well did you know this?
1
Not at all
2
3
4
5
Perfectly
12
Q

A woman made a will leaving a gift of “£50,000 to the children of my sister who reach the age of 18”. The woman died three months ago and was survived by her sister who has two children, aged 20 and 14. The sister now discovers that she is pregnant with a third child, conceived after the woman’s death.

Which of the following best describes how this gift will be distributed?

A The gift of £50,000 will pass only to the 20-year-old.

B The gift of £50,000 may pass to both the 20-year-old and the 14-year-old.

C The gift of £50,000 may pass to the 20-year-old, the 14-year-old, and the baby.

D The gift of £50,000 will initially pass to the 20-year-old and the 14-year-old but will be redistributed on each occasion that the sister has another child.The gift of £50,000 will initially pass to the 20-year-old and the 14-year-old but will be redistributed on each occasion that the sister has another child. - no response given

E The gift will fail for uncertainty.

A

The gift of £50,000 may pass to both the 20-year-old and the 14-year-old.

(B) The gift of £50,000 may pass to both the 20-year-old and the 14-year-old. A class gift is a gift of property to be divided among beneficiaries who fulfil a general description. Here, the class gift is to be divided among the children of the sister who reach the age of 18. Class closing rules apply to determine how and when a class gift should be distributed. Generally, a class closes – to the exclusion of any potential beneficiary not then living-when at least one beneficiary has a vested interest. When there is a contingent gift, with a condition that needs to be satisfied, the class closes at the date of the testator’s death if there is any living beneficiary who has met the condition. The class then includes any other living beneficiary who later meets the condition. Here, the class closes at the date of the woman’s death because the sister already has one child who has reached the age of 18 and has therefore fulfilled the condition. Any future children of the sister who are ‘living’ at this time may also claim a share of the £50,000 when they reach the age of 18. This includes the 14-year-old only. The baby has no future entitlement as it had not been conceived before the class closed on the woman’s death. (A) is incorrect. Whilst the 20-year-old has fulfilled the contingency and so can claim a share of the £50,000 now, the 14-year-old can also claim a share when they reach the age of 18. (C) is incorrect. The baby was not conceived when the class closed on the woman’s death and so is not entitled to a share. (D) is incorrect. The class closed on the woman’s death, and any future children of the sister will have no entitlement to share in the £50,000. (E) is incorrect. A class gift such as this is valid.

How well did you know this?
1
Not at all
2
3
4
5
Perfectly
13
Q

A man has owned 7,500 shares in an unlisted trading company since January 2001. On 1 July 2023, he gave 3,000 shares to his son. The company has an issued share capital of 10,000 shares.

The values of different shareholdings in the shares on 1 July 2023 are as follows:

Up to 25%: £5

26% to 50%: £8

51% to 74%: £13

75% or more: £20

What is the transfer of value for inheritance tax purposes on the gift of the shares to his son?

A £0 under the related property rule

B £24,000

C £60,000

D £58,500

E £114,000

A

£114,000.

£114,000. The transfer of value is always the loss to the donor. With unquoted shares the value by which the estate has diminished is used. The value of the man’s shares before the gift was £150,000 (since he owned 75% of the shares, we calculate value at £20 per share). After the gift, the man held 45% of the shares (4,500) which were worth £8 per share (£36,000). £150,000 - £36,000 = £114,000

How well did you know this?
1
Not at all
2
3
4
5
Perfectly
14
Q

A father paid £50,000 towards his daughter’s purchase of an apartment costing £100,000. The apartment was conveyed into the daughter’s name alone, and there was no declaration or evidence as to the father’s intentions. The father and daughter are now estranged, and the daughter has sold the flat for £150,000. The father wishes to claim as much as possible from the sale proceeds.

Which of the following best describes the father’s claim?

A He may reclaim £50,000 under a presumed resulting trust.

B He has no claim because a gift is presumed.

C He may claim £75,000 if he can prove that no gift was intended.

D He may claim £75,000 under a presumed resulting trust.

E He may reclaim £50,000 if he can prove that no gift was intended.

A

He may claim £75,000 if he can prove that no gift was intended.

(C) The father may claim £75,000 if he can prove that no gift was intended. Where an individual contributes to the purchase of property in the name of another and there is no evidence that a gift was intended, the usual presumption is that the legal owner holds on resulting trust for herself and the other party in proportion to their respective contributions. This presumption does not apply where the contributor was the father or husband of the legal owner, or was acting in loco parentis to the legal owner. In these cases the presumption of advancement applies and it is presumed that the contributor intended to make a gift unless he can prove that he did not. Here, the presumption of resulting trust does not apply because the contributor was the legal owner’s father. It is therefore presumed that he intended a gift unless he can prove that he did not. (A) and (D) are incorrect because the presumption of resulting trust does not apply here as the contributor was the father of the legal owner. (B) is incorrect because, although the presumption is that a gift was intended, the presumption can be rebutted by evidence that this was not the case. (E) is incorrect because, if it can be shown that the daughter holds on resulting trust, she holds in proportion to their contributions, so the father would be entitled to claim half of the sale proceeds.

How well did you know this?
1
Not at all
2
3
4
5
Perfectly
15
Q

A man made a valid will 10 years ago. It included the following provisions:

(1) I give my LMN plc shares to my cousin’s two daughters jointly.

(2) I give the remainder of my estate to my sister.

There are no other relevant clauses. The man has just died. The cousin’s older daughter died last year, but the other daughter survives. The man’s sister is also still alive.

Which of the following best describes entitlement to the man’s estate?

A The sister will inherit the entire estate.

B The cousin’s surviving daughter will inherit the LMN plc shares and the sister will inherit the remainder of the estate.

C The cousin’s surviving daughter will receive half of the shares, and the other half of the shares will pass to the man’s sister. The man’s sister will also inherit the rest of his estate.

D The cousin’s surviving daughter will receive half of the shares, and the other half will pass via the intestacy rules. The man’s sister will inherit the remainder of the estate.

E The shares will pass via the intestacy rules, and the man’s sister will inherit the remainder of the estate.

A

The cousin’s surviving daughter will inherit the LMN plc shares and the sister will inherit the remainder of the estate.

(B) The cousin’s surviving daughter will inherit the LMN plc shares and the sister will inherit the remainder of the estate. If a beneficiary has predeceased the testator, the gift to them will lapse – that is, fail. A gift to two or more people as joint tenants will not lapse unless all the recipients die before the testator. If one joint tenant dies, the surviving joint tenants are entitled to the entire gift. Here, the man left his shares to his cousin’s two daughters jointly. Because the man gave the gift to the daughters as joint tenants, the surviving daughter is entitled to all of the shares. (A), (C), (D), and (E) are, therefore, incorrect. The surviving daughter will inherit all of the shares, and the intestacy rules will not apply.

How well did you know this?
1
Not at all
2
3
4
5
Perfectly
16
Q

A buyer of a piece of land recently completed his purchase. A deed of easement over the piece of land was included in the epitome of title. The buyer claims that he did not know about the right of way contained in the deed and that he is not bound by the right of way.

Which of the following best describes the buyer’s position?

A The buyer is not bound by the right of way because he did not know about it at completion.

B The buyer is bound by the right of way if the easement was registered as a D(iii) land charge.

C The buyer is bound by the right of way because he had imputed notice of it.

D The buyer is bound by the right of way because it is a legal interest and he is bound irrespective of notice.

E The buyer is not bound by the right of way because he is a bona fide purchaser for valuable consideration without notice of it.

A

The buyer is bound by the right of way because it is a legal interest and he is bound irrespective of notice.

(D) The buyer is bound by the right of way because it is a legal interest and he is bound irrespective of notice. A buyer of a legal estate will purchase subject to any legal interests which subsist whether or not they were aware of them. Legal interests bind a buyer irrespective of notice. A legal easement is one that is created, as here, by deed. Thus, the buyer is bound by this legal interest irrespective of notice. (A) is incorrect because, as explained above, the buyer is bound by the legal easement irrespective of notice. (B) is incorrect because a D(iii) land charge protects an equitable easement, and this is a legal easement. (C) and (D) are incorrect because imputed notice (notice given to an agent acting on behalf of the buyer) and the buyer’s position as a bona fide purchaser are relevant to the doctrine of notice, which is used to determine the enforceability of certain equitable interests. The easement here is a legal interest

How well did you know this?
1
Not at all
2
3
4
5
Perfectly
17
Q

A man is buying a house which is 20 years old. The survey he commissioned includes the following information:

‘Internal works have been recently carried out to the property. A wall was erected in the large dining space to create an office room in the rear of the original room. From my inspection, the internal works were minor in nature and did not involve alteration to any load bearing walls.’

The seller did not obtain permission from the local authority to undertake the work. Though the surveyor’s report confirmed the works were minor internal works, the man is still concerned and discusses the matter with his solicitor.

What advice is the solicitor likely to give regarding this issue?

A That the work has been carried out to a listed building and so planning permission was required for the work.

B That the work is considered to be development and thus the seller should have obtained planning permission.

C That the work is not considered to be development and thus planning permission was not required.

D That the work constitutes a material change of use and thus the seller should have obtained planning permission.

E That the work is considered to be development but nonetheless did not require express permission because it could be carried out under the permitted development regime.

A

That the work is not considered to be development and thus planning permission was not required.

(C) The solicitor should advise the client that the work is not considered to be development and thus planning permission was not required. Under the Town and Country Planning Act 1990, planning permission is required for any development of land. Development includes building and making a material change of use. However, minor internal work is not considered to be development and so planning permission is not required. The survey confirms that the work was minor in nature. Therefore, (B) is incorrect. (A) is incorrect because while any alterations to a listed building (that is, one listed as historical or otherwise significant) usually need planning permission, there is no suggestion in the facts that the property here is a listed building. Additionally, the facts indicate the property is a house that is only 20 years old. It is highly unlikely to be on the list. (D) is incorrect because merely creating two rooms from one internally would not constitute a material change of use. Change of use contemplates changes such as conversion of one house to two flats or a house to a shop and a flat. (E) is incorrect because if work is not considered to be development in the first place, then deemed permission under the permitted development regime is irrelevant (because no permission is required)

How well did you know this?
1
Not at all
2
3
4
5
Perfectly
18
Q

Question
A store owner is granted a 40-year lease on a shop on 1st January 2023. The premium payable is £55,000 and the net present value of the rent payable is £175,000. Annual rents under the lease are £6,000 per annum (a total of £240,000).

On what amount will the stamp duty land tax be calculated?

A £0, as stamp duty land tax is not owed on leases.

B £55,000, as stamp duty land tax will be calculated based only on the premium paid.

C £175,000, as stamp duty land tax will be calculated based only on the present value of the lease payments.

D £240,000, as stamp duty land tax will be calculated based only on the actual value of the rent to be paid.

E £230,000, as the stamp duty land tax will be calculated based only on both the premium paid and the present value of the lease payments.

A

£230,000, as the stamp duty land tax will be calculated based only on both the premium paid and the present value of the lease payments.

How well did you know this?
1
Not at all
2
3
4
5
Perfectly
19
Q

A landlord owns an office building. Several floors of the building are only partially filled. A potential tenant has offered to lease space in the building if the landlord can lease an entire floor of the building to the tenant. The third floor of the building has only one tenant who leases half the floor. That tenant’s lease will expire in eight months. So that she may lease the third floor to the prospective tenant, the landlord would like to terminate the lease of the current tenant at the end of its term and offer that tenant similar office space on the fourth floor of the building.

The landlord and current tenant did not contract out of the provisions of the Landlord and Tenant Act 1954 (Part II) (‘1954 Act’) when they entered the lease that is about to expire.

May the landlord terminate the tenancy at the end of the lease term under these circumstances?

A No, because the time for a section 26 notice has already passed.

B Yes, because the landlord will offer suitable alternative accommodations to the tenant.

C No, because none of the statutory grounds for possession under the 1954 Act apply.

D Yes, because the contractual term of the lease will have ended.

E No, because the time for a section 25 notice has already passed.

A

Yes, because the landlord will offer suitable alternative accommodations to the tenant.

(B) The landlord may terminate the lease because the landlord will offer suitable alternative accommodations to the tenant. Under the 1954 Act, a tenant can continue to occupy leased commercial premises under the lease after the term of the lease expires unless the landlord serves a section 25 notice on the tenant between six to 12 months before the end of the lease term. The notice must state a statutory ground for termination of the lease. Such grounds include breaches of obligations under the lease by the tenant; the availability of suitable, alternative premises for the tenant; and the landlord’s desire to demolish, reconstruct, or move into the premises. Here, the landlord will offer suitable alternative premises to the tenant (making (C) incorrect). Additionally, the facts indicate that there are eight months left on the lease, so the time for a section 25 notice has not expired (making (E) incorrect). (A) is incorrect, both because the time for a notice to terminate the tenancy has not passed and because the notice from the landlord is a section 25 notice (a section 26 notice is a notice from a tenant to request a new lease at the end of the lease term). (D) is incorrect because merely reaching the end of the term of a lease is not a ground for terminating a commercial tenancy under the 1954 Act.

How well did you know this?
1
Not at all
2
3
4
5
Perfectly
20
Q

A woman died intestate, survived by her husband and her daughter from a previous relationship. The assets in her sole name totalled £700,000, comprising the family home (£450,000), personal belongings (£50,000), and £200,000 in shares.

Which of the following best describes how the woman’s estate will be distributed?

A The woman’s daughter will be entitled to a statutory legacy of £322,000 only.

B The woman’s daughter will have no entitlement to the woman’s estate, and the whole estate will pass to the husband.

C The woman’s estate will be split so that the husband and daughter receive half each.

D The daughter will be entitled to the sum of £164,000 from the estate.

E The personal belongings and family home will pass to the husband, whilst the shares will be divided equally between the husband and the daughter.

A

The daughter will be entitled to the sum of £164,000 from the estate.

(D) The daughter will be entitled to the sum of £164,000 from the estate. The rules of intestate succession apply when a person dies without a will. Under these rules, when the deceased is survived by a spouse or civil partner and issue, the spouse or civil partner will receive personal chattels, £322,000, and one-half of the residue. The deceased’s issue will take the other half of the residue. The intestacy rules classify children born to unmarried parents as issue, and so the daughter is entitled to share in the estate. The husband will receive the personal belongings (worth £50,000), a statutory legacy of £322,000, and half of the remaining assets. Once the personal belongings and statutory legacy are received by the husband, this leaves £328,000 of the £700,000 estate remaining. This sum is split equally between the husband and daughter, and so the daughter receives £164,000 under the intestacy rules. (A) is incorrect. It is the spouse, not the daughter, who is entitled to the statutory legacy. (B) is incorrect. Under the intestacy rules, the daughter is entitled to share in the estate. It does not matter that her parents were unmarried. (C) and (E) are incorrect as they do not accurately describe how the estate will be shared in this intestacy situation.

How well did you know this?
1
Not at all
2
3
4
5
Perfectly
21
Q

Question
A man died three weeks ago, leaving a will in which his civil partner was appointed as his sole executor. The executor is keen to protect himself against claims from unknown creditors. He now plans to place advertisements to publicise the man’s death in the London Gazette, a local newspaper, and a few other appropriate publications.

How long should the man’s civil partner wait before distributing the estate to protect himself against claims from unknown creditors as a result of these advertisements?

A Two months from the date of the man’s death.

B Six months from the date of the man’s death.

C Six months from the date of the grant of probate.

D Two months from the date of the advertisements.

E Two months from the date of the grant of probate.

A

Two months from the date of the advertisements.

(D) Under section 27 Trustee Act 1925, an executor can receive protection from claims by unknown creditors by advertising the decedent’s death in the London Gazette, a local newspaper, and any other appropriate newspaper. They then must wait two months from the date of the advertisements before distributing the estate. Consequently, the time periods stated in (A), (B), (C), and (E) are incorrect.

How well did you know this?
1
Not at all
2
3
4
5
Perfectly
22
Q

A suspect is arrested on suspicion of involvement in a drugs cartel importing class A drugs into the UK. The officer in charge of the investigation wishes to prevent the defendant from having legal advice. The officer suspects exercise of the right to legal advice will result in other members of the cartel being alerted to the investigation and the consequent destruction of evidence.

Can the suspect be refused access to legal advice?

A Yes, access to legal advice can be delayed for a maximum of 24 hours.

B Yes, access to legal advice can be delayed for a maximum of 36 hours.

C Yes, access to legal advice can be refused entirely.

D No, access to legal advice can be refused only if the refusal is likely to prevent an act of violence.

E No, every person under arrest has the right to receive private legal advice during detention at any time.

A

Yes, access to legal advice can be delayed for a maximum of 36 hours.

(B) Access to legal advice can be delayed for 36 hours. Delay in accessing legal advice is permitted only when: (1) the suspect is arrested on an indictable only or either way offence, (2) a police officer of the rank of superintendent or above has authorised the delay in writing, and (3) the officer has reasonable grounds to believe that exercise of the right will lead to interference with evidence, interference with others, alerting other suspects, or hindering the recovery of property related to the offence. The right can be delayed 36 hours at most, and so (B) is the correct answer. (E) is incorrect as access to legal advice can in some circumstances be prevented. (A) is incorrect as it misstates the length of time legal advice can be delayed. (C) is incorrect as access to legal advice can be delayed for a maximum of 36 hours. (D) is incorrect because access to legal advice can be delayed for other reasons as stated above

How well did you know this?
1
Not at all
2
3
4
5
Perfectly
23
Q

A man approached the cashier at a local petrol station and offered to exchange a pair of sunglasses for some petrol. The cashier refused. The man then pulled a knife out of his pocket and told the cashier he wanted to fill up. The cashier, who was quite a bit older than the man, gave him some ‘fatherly advice’ that crime does not pay. In response to the advice, the man put the knife away. Feeling sorry for the man, the cashier then agreed to give the man some petrol for the sunglasses. The man then left. The cashier then discovered the man had taken the sunglasses from a display case in the store and clipped the tag off before offering them to the cashier. The man was arrested shortly thereafter.

With which offences should the prosecution charge the man?

A The man should be charged with theft and attempted robbery.

B The man should be charged with theft and robbery.

C The man should be charged with fraud by false representation and attempted robbery.

D The man should be charged with theft but not attempted robbery because he voluntarily abandoned the attempt.

E The man should be charged with fraud by false representation but not with attempted robbery because he voluntarily abandoned the attempt.

A

The man should be charged with fraud by false representation and attempted robbery.

(C) The man should be charged with fraud by false representation and attempted robbery. Fraud by false representation requires that the defendant dishonestly make a false representation intending to make a gain for himself or a loss for another. Here the man dishonestly represented that the glasses were his, intending to make a gain for himself (the petrol). The man can also be convicted of attempted robbery because he attempted a taking of the property of another in the presence of the victim by force and with the intent to permanently deprive the victim of it. The fact that the man was persuaded not to carry out the robbery does not affect his liability for attempt; that crime was completed as soon as he pulled out a knife and demanded the petrol. He did an act that was more than merely preparatory. However, he did not actually obtain the petrol by use of force. Therefore, he did not commit robbery, so (B) is incorrect. (A) and (D) are incorrect because theft occurs when the man intends to deprive the cashier of the glasses permanently. Here, the man just wanted to use the glasses as a way to convince the cashier to give him petrol. It is arguable that the man’s actions were analogous to acting as the owner by clipping off the tag. However, fraud by misrepresentation is more clearly made out and, therefore, a far more preferable charge to theft. (E) is incorrect because the rule is that abandonment is not a defence to attempt. As discussed above, the crime of attempted robbery was completed as soon as the man pulled the knife out of his pocket and demanded petrol

How well did you know this?
1
Not at all
2
3
4
5
Perfectly
24
Q

A defendant is charged with theft of a case of wine worth £100 from a grocery store. The defendant plans to plead not guilty.

Where will the case be heard?

A Magistrates’ Court.

B Magistrates’ Court or Crown Court, depending on where the defendant elects trial.Magistrates’ Court or Crown Court, depending on where the defendant elects trial.

C Magistrates’ Court or Crown Court, depending on whether the Magistrates’ Court accepts jurisdiction.

D Magistrates’ Court or Crown Court, depending on whether the Magistrates’ Court accepts jurisdiction and where the defendant elects trial.

E Crown Court.

A

Magistrates’ Court or Crown Court, depending on where the defendant elects trial.

(B) The case can be heard in the Magistrates’ Court or Crown Court, depending on where the defendant elects trial. Whilst theft is ordinarily an either way offence, theft under the value of £200 is treated as summary only in that the Magistrates’ Court cannot decline jurisdiction. However, the defendant is able to elect Crown Court trial. (A) is incorrect, as the defendant is still able to elect Crown Court trial. (C) and (D) are incorrect, as the Magistrates’ Court cannot decline jurisdiction. (E) is incorrect, as the matter will stay in the Magistrates’ Court if the defendant accepts

How well did you know this?
1
Not at all
2
3
4
5
Perfectly
25
Q

A minor beneficiary has an absolute vested interest under a trust of which they are the sole beneficiary. The beneficiary wishes to use the income for the payment of school fees.

Which of the following best states the legal position with regard to such use?

A The trustees must give the minor beneficiary income for the payment of school fees because the beneficiary has a vested interest.

B The trustees must pay the minor beneficiary’s school fees, but the income must be paid directly to the school or to a parent or guardian.

C The trustees may, at their discretion, give income to the minor beneficiary to pay for the school fees.

D The trustees must give the minor beneficiary income for the payment of school fees if the beneficiary can demonstrate genuine financial need.

E The trustees may, at their discretion, pay the minor beneficiary’s school fees, but they must give the income directly to the school or to a parent or guardian.

A

The trustees may, at their discretion, pay the minor beneficiary’s school fees, but they must give the income directly to the school or to a parent or guardian.

(E) Under the power of maintenance, trustees have the power to pay or apply the income of the trust for a minor beneficiary’s maintenance, education, or benefit. The power is discretionary. Because a minor is not capable of giving a valid receipt, the income must be applied directly to the purpose required or to a parent or guardian for use in the beneficiary’s interest. Therefore, the trustees may, at their discretion, pay the minor beneficiary’s school fees, but they must give the income directly to the school or to a parent or guardian. (A) and (B) are incorrect because the power of maintenance is discretionary and applies to minor beneficiaries with vested or contingent interests. (C) is incorrect because the trustees must give the income to the school or to a parent or guardian, not to the minor beneficiary. (D) is incorrect because the power of maintenance is discretionary and because there is no requirement for the beneficiary to demonstrate financial need.

How well did you know this?
1
Not at all
2
3
4
5
Perfectly
26
Q

A 20-year-old woman was arrested and charged with manslaughter.

In what court will the case against the woman be commenced?

A The Crown Court.

B The Criminal Court.

C The County Court.

D The Justice Court.

E The Magistrates’ Court.

A

The Magistrates’ Court.

(E) Prosecution of all criminal offences, including murder, starts in the Magistrates’ Court. The magistrate is then obligated to transfer the murder case to the Crown Court. Other offences such as theft also start in the Magistrates’ Court, but they need not be sent to the Crown Court unless they are sufficiently serious to warrant greater punishment than the Magistrates’ Court can impose. Therefore, (A), (B), (C), and (D) are incorrect.

How well did you know this?
1
Not at all
2
3
4
5
Perfectly
27
Q

A defendant is charged with shoplifting. The allegation is that the defendant stole makeup worth £100.00 from the local supermarket.

In what court will the criminal prosecution for this offence begin?

A The Magistrates’ Court, as all prosecutions of adults begin in the Magistrates’ Court.

B The Magistrates’ Court, as low value shoplifting is considered summary only for the purposes of allocation and so cannot be heard in the Crown Court.

C The Crown Court, as high value shoplifting is considered indictable only and so will go to the Crown Court.

D The Magistrates’ Court, as high value shoplifting is considered an either way offence and so trial will take place in the Magistrates’ Court.

E Either the Magistrates’ Court or the Crown Court, as it is an either way offence and the defendant can elect where they have their first appearance.

A

The Magistrates’ Court, as all prosecutions of adults begin in the Magistrates’ Court.

(A) All prosecutions, for summary only, either way, and indictable only offences, start in the Magistrates’ Court. Shoplifting under £200 is considered low value theft, and so the Magistrates’ Court cannot decline jurisdiction (though the defendant can still elect Crown Court trial if they prefer). However, the issue here is that all first appearances for all offences take place in the Magistrates’ Court, and so (B), (C), (D), and (E) are all incorrect.

How well did you know this?
1
Not at all
2
3
4
5
Perfectly
28
Q

The managing director of a law firm’s client disputes a bill the firm recently issued for professional services. The firm agrees to reduce the professional fees by £500 to £3,500.

How should this reduction be reflected in the client ledger?

A In the client ledger account, credit the client side with £500.

B In the cash account, credit the client side with £500.

C In the client ledger account, debit the business side with £500 and £100 to reflect the reduction in VAT.

D In the client ledger account, credit the business side with £500 and £100 to reflect the reduction in VAT.

E In the client ledger account, delete the original entry and re-enter the correct sums.

A

In the client ledger account, credit the business side with £500 and £100 to reflect the reduction in VAT.

(D) The reduction will be shown as a credit on the business side of the client account (and a debit in the profit cost and HMRC ledgers) to reflect the reduction in costs actually billed. Thus (D) is correct. (A) is incorrect because it should be a credit on the business side. (B) is incorrect because the question specifically asked about the client ledger account rather than the cash account. (C) is incorrect because the reduction should be shown as a credit to the client and not as a debit. (E) is incorrect because the accounts must accurately track the sequence of events, including the issuing of the original invoice and the subsequent reduction in fees.

How well did you know this?
1
Not at all
2
3
4
5
Perfectly
29
Q

The title to Green Farm is registered at His Majesty’s Land Registry. There are three registered charges on the charges register: the first registered in February 2013, the second registered in March 2014 and the third registered in October 2015. All three charges are residential mortgages using standard mortgage deeds. The registered proprietor has stopped making payments to the third lender but has kept up his payments on the other two mortgages.

Which of the following best describes the position of the third lender?

A It can take possession proceedings only if it provides notice to the first and second lenders and both decline to take possession proceedings themselves.It can take possession proceedings only if it provides notice to the first and second lenders and both decline to take possession proceedings themselves. - incorrect

B It can take possession proceedings and can retain the sale proceeds in their entirety to pay off its charge.

C It can take possession proceedings but will have to discharge the mortgages in favour of the first lender and the second lender out of the sale proceeds before it can pay off its own mortgage.

D It can take possession proceedings and repay its own charge out of the sale proceeds first and then must pass on the balance of the sale proceeds to the defaulting registered proprietor.

E It can take possession proceedings and will have to split the sale proceeds equally between all three lenders based on the principles of equity.

A

It can take possession proceedings but will have to discharge the mortgages in favour of the first lender and the second lender out of the sale proceeds before it can pay off its own mortgage.

(C) The third lender can take possession proceedings but will have to discharge the first and second mortgages before it can pay off its own mortgage. When a legal mortgage is entered on the register of title, it will have priority over any interest whose priority is not protected when the mortgage is registered. Here, the mortgages have priority in order of their registration on the register of title. Thus, due to the doctrine of priority of mortgages, the third lender must pay off the prior charges of the first and second lenders out of the sale proceeds before its own. (A) is incorrect because the third lender may take possession proceedings without notifying or allowing the first and second lenders to do so first. However, as explained above, the third lender must discharge the first and second lenders’ mortgages out of the sale proceeds before paying off its own mortgage. (B) and (D) are incorrect because, as explained above, the third lender cannot pay itself first out of the sale proceeds due to the doctrine of priority of mortgages. The third lender must apply the sale proceeds to the repayment of the prior charges in the order that they were registered. If there are any sale proceeds left once the first and second lenders have been paid off, this will be paid to the third lender, and any balance remaining will then go to the defaulting registered proprietor. (E) is incorrect because priority of mortgages requires that the sale proceeds are applied in the order that the charges were registered, with the first lender being first in line

How well did you know this?
1
Not at all
2
3
4
5
Perfectly
30
Q

A defendant was arrested on suspicion of criminal damage. He was interviewed under caution at the police station and answered questions, stating that he was at home at the time of the offence. At trial, on advice from his legal representative, the defendant does not give evidence.

Can an adverse inference be drawn from the defendant’s silence?

A Yes, as the defendant is compellable for the defence.

B Yes, as the defendant has not given an account and been subjected to cross-examination.

C No, as the defendant was following the advice of his solicitor.

D No, as the defendant gave an account in police interview.

E No, as the defendant has no obligation to give evidence at trial.

A

Yes, as the defendant has not given an account and been subjected to cross-examination.

(B) An adverse inference can be drawn from the defendant’s silence because he has not given evidence at trial and been subjected to cross-examination. The defendant is not compellable for either the prosecution or the defence, and so (A) is incorrect. However, a failure to give evidence and be exposed to cross-examination can result in an adverse inference being drawn. (C) is incorrect because legal advice to remain silent will not preclude the drawing of an adverse inference. (D) is wrong as an inference can be drawn from a failure to give evidence at trial, even when an account was provided in interview. (E) is wrong as, whilst there is no obligation to give evidence at trial, a failure to do so can lead to an adverse inference being drawn.

How well did you know this?
1
Not at all
2
3
4
5
Perfectly
31
Q

In his will a man gave £100,000 to trustees on trust to invest and use the income for the maintenance of the building of the Newtown Museum and Art Gallery. Shortly after his death, the museum closed, the building was sold, and the contents of the museum were dispersed.

Which of the following statements best describes the effect of the gift?

A The gift will be applied cy-pres if the court finds evidence of general charitable intention.

B The trust fails and the trustees hold on resulting trust for the residue of the man’s estate.

C The gift will be applied cy-pres for a similar purpose.

D The trustees have discretion to use the fund for a similar purpose.

E The trust fails because its terms breach the inalienability rule.

A

The gift will be applied cy-pres for a similar purpose.

(C) The gift will be applied cy-pres for a similar purpose. Where a trust for a charitable purpose has taken effect but subsequently fails because it is impossible or impractical to carry out, the funds will be applied cy-pres to a similar charitable purpose. This is a charitable trust because it is for a charitable purpose, it is for the public benefit, and it is exclusively charitable. The gift vested at the date of the man’s death, at which time the museum was still in existence. This means that the gift for a charitable purpose was effective at the date of death, and the funds will be applied cy-pres. (A) is incorrect because it is only necessary to find evidence of general charitable intention in a case of initial failure. (B) is incorrect because the trust does not fail. (D) is incorrect because the application of the fund is not for the trustees to decide. (E) is incorrect because the inalienability rule does not apply to charitable trusts.

How well did you know this?
1
Not at all
2
3
4
5
Perfectly
32
Q

A solicitor is acting for a woman applying for first registration of her unregistered title. She possesses the title deeds to the property and all necessary documentation. The woman mentions to her solicitor that she recently granted a five-year lease to a tenant who is currently occupying the property.

What class of title is likely to be granted to the woman when the property is registered?

A Freehold title.Freehold title.

B Good freehold title.

C Absolute title.

D Possessory title.

E Leasehold title.

A

Absolute title.

(C) On first registration, His Majesty’s Land Registry (‘HMLR’) is likely to grant absolute freehold title since the woman possesses the title deeds and all necessary documentation as to her ownership. The tenant’s lease may be an overriding interest (as a legal lease for seven years or less), meaning the woman’s registered estate will be subject to it, but the existence of the lease does not mean the woman will not be awarded absolute title. (A) is incorrect because it is not a class of title. A freehold estate is one of the two legal estates in land in England and Wales. (B) is incorrect because this is not a class of title. Good leasehold title may be awarded when the freehold title has not been produced to HMLR on application to register a lease. (D) is incorrect because possessory title is a class of title based on factual possession of the land rather than documentary evidence. It is commonly given when an application for registration is based on adverse possession or when the title deeds have been lost. Here, the woman possesses the title deeds and necessary documentation, so possessory title will not be awarded. (E) is incorrect because this is not a class of title. A leasehold estate is the other of the two legal estates in land in England and Wales.

How well did you know this?
1
Not at all
2
3
4
5
Perfectly
33
Q

A defendant appears before court charged with theft. The district judge believes that a custodial sentence is likely and is minded to refuse bail.

Which of the following best states an exception to the right to bail upon which the defendant could be refused bail?

A The defendant was previously charged with battery and was on bail at the time of the offence.

B There are grounds to believe that the defendant would fail to surrender.

C There are grounds to believe that the defendant would commit further offences on bail.

D There are substantial grounds to believe that the defendant would interfere with witnesses.

E There are grounds to believe that the defendant would obstruct justice.

A

There are substantial grounds to believe that the defendant would interfere with witnesses.

(D) Bail can be refused if there are substantial grounds to believe that the defendant would fail to surrender, commit further offences, or interfere with witnesses or otherwise obstruct justice. (B), (C), and (E) are all incorrect as they miss the key term substantial. Bail can also be refused if the defendant is charged with an offence that can be tried in the Crown Court (such as burglary, arson, or GBH), and they were on bail at the time of the offence. Whilst in (A) the defendant was previously charged with battery, this offence cannot be tried in the Crown Court because it is summary only, and so (A) is also incorrect

How well did you know this?
1
Not at all
2
3
4
5
Perfectly
34
Q

A woman asked her friend if she could borrow his car to drive to the grocery store. Although the friend knew the woman sometimes sold illegal drugs, he gave the woman his car keys and told her she could use the car to do her shopping. The woman drove to the store as agreed because that is where she had arranged to meet someone to sell them illegal drugs. The woman was arrested as soon as she passed the illegal drugs to the buyer. The police then traced the car back to the woman’s friend.

Can the friend be held criminally liable for the sale of the illegal drugs?

A The friend can be held criminally liable for the sale of the illegal drugs as an accomplice.

B The friend cannot be held criminally liable for the sale of the illegal drugs because the friend’s acts were not sufficient for liability for aiding, abetting, procuring, or counselling.

C The friend can be held criminally liable for the sale of the illegal drugs because the friend knew the woman sometimes sold illegal drugs.

D The friend cannot be held criminally liable for the sale of the illegal drugs because supplying transportation is not part of the actus reus of illegally selling drugs.

E The friend can be held criminally liable for the sale of the illegal drugs because the friend’s actions were sufficient to impose liability on him as a co-principal.

A

The friend cannot be held criminally liable for the sale of the illegal drugs because the friend’s acts were not sufficient for liability for aiding, abetting, procuring, or counselling.

(B) The friend is not guilty of any crime because the friend’s acts were not sufficient for liability for aiding, abetting, procuring, or counselling. The best way to arrive at this choice is to eliminate the other choices. (A) is incorrect. An accomplice aids, abets, counsels, or procures another offender. Here, despite the fact the friend knew the woman sometimes sold illegal drugs, nothing indicates he thought the woman was going to the store for any purpose other than to buy groceries. Therefore, he did not act as an aider, abettor, etc. (C) is incorrect. The mere knowledge that a person sometimes commits crimes is not a sufficient basis for imposing criminal liability on anyone who unknowingly helps the person commit a crime. (D) is incorrect because a person can be guilty of aiding even if the person’s action is not part of the physical acts (actus reus) required for the crime. Any act that aids, abets, procures, or counsels can be a sufficient basis for imposing liability. (E) is incorrect because a co-principal is one who acts together with another to commit the prohibited act, and here the friend did not act with the woman in selling the illegal drugs.

How well did you know this?
1
Not at all
2
3
4
5
Perfectly
35
Q

A man is arrested on Monday at 2pm on suspicion of robbery. He enters the police station at 3pm. He is signed into the custody suite at 4pm.

Until when can the man be kept in custody without charge if all possible extensions of time are granted?

A Thursday 2pm.

B Thursday 3pm.

C Friday 2pm.

D Friday 3pm.

E Friday 4pm.

A

(D) The man can be kept in custody without charge until Friday 3pm. The maximum time that a suspect can remain in custody prior to charge is 96 hours from the ‘relevant time’. The relevant time is when the suspect arrives at the police station. The relevant time here is 3pm on Monday. 96 hours later is 3pm on Friday, and so (D) is the correct answer. Accordingly, (A), (B), (C), and (E) are incorrect because they use an incorrect relevant time or an incorrect detention period.

How well did you know this?
1
Not at all
2
3
4
5
Perfectly
36
Q

A man has instructed a solicitor to act for him on his purchase of a coastal property. The man is concerned about whether there are any public rights of way over the property and asks the solicitor to confirm that there are none.

How will the solicitor find out the information which the client is seeking?

A The drainage and water search.

B The local search.

C The commons registration search.

D The environmental search.

E The coal mining search.

A

(B) A local search gathers information from the local authority. The first part of a local search reveals whether there are registrations by the local authority in the land charges register. The second part is a standard set of enquiries of the local authority which include enquiries about roads fronting the property, public rights of way, and planning entries and building regulations relevant to the property. A local search may also include optional enquiries. (A) is incorrect because, as the name implies, a drainage search is an enquiry sent to the local water company that asks whether the property is connected to a public foul drainage system and public water supply. (C) is incorrect because a commons registration search is an optional search which is performed when the property appears to abut common land or a town or village green to determine whether use of the property is restricted or burdened by its proximity to public land. (D) is incorrect because an environmental search is an electronic database search that a solicitor may perform from their office to determine whether the property is likely to have been contaminated by hazardous wastes in the past (because if it was, the buyer could be liable for clean-up costs in the future). (E) is incorrect because, as you might expect, a coal mining search is performed in coal mining areas to discover whether the property might be impacted by a coal mine.

How well did you know this?
1
Not at all
2
3
4
5
Perfectly
37
Q

A defendant is standing trial on an allegation of grievous bodily harm (‘GBH’) on his ex-girlfriend in the Crown Court. The defendant is giving evidence. He states, “I was acting in self-defence. That bad girl is violent. She beat me and I had to fend her off. She’s been done for it before; it’s just what she’s like”.

What will be the effect of the defendant’s statement about his ex-girlfriend?

A Evidence of the defendant’s bad character remains inadmissible because his statement falls outside the seven gateways of admissibility.

B Evidence of the defendant’s bad character will be admissible only if it is relevant to an important matter in issue between the prosecution and the defence.

C Evidence of the defendant’s bad character will be admissible only if it is important explanatory evidence.

D Evidence of the defendant’s bad character will be admissible in the proceedings.

E Evidence of the defendant’s bad character will be admissible in the proceedings unless the defendant did not realise this would be the consequence.

A

Evidence of the defendant’s bad character will be admissible in the proceedings.

(D) The defendant’s statement about his ex-girlfriend will cause evidence of his bad character to be admissible in the proceedings. One of the seven gateways to bad character being admitted in evidence is when the defendant attacks the character of another, which is the case here. There is no requirement for the defendant to realise the effect of their statement, and so (E) is incorrect. (A) is incorrect as the defendant’s statement falls under the gateway of the defendant attacking another’s character. (B) and (C) are incorrect as, whilst evidence of the defendant’s bad character could potentially also be admissible under the grounds cited, these are not the only routes to admission because the defendant clearly attacked his ex-girlfriend’s character

How well did you know this?
1
Not at all
2
3
4
5
Perfectly
38
Q

An executor is administering the estate of a woman who died recently. The woman had a son, currently aged 18, who has an interest in the estate contingent upon him reaching the age of 21. The executor will continue to hold the son’s interest (comprising a sum in a high interest bank account) on trust in the meantime. The will contains no provisions in relation to the holding of this sum.

The son has asked for the interest generated on the bank account to be paid to him now and going forward.

Which of the following best states the position of the executors in relation to this request?

A The executor must accumulate the interest received in the bank account and cannot pay it to the son until he is 21.

B The executor is required to pay all interest to the son now, as he has reached the age of 18.

C The executor retains full discretion about whether to pay the interest to the son.

D The executor can choose to apply the interest only for purposes related to the son’s maintenance, education, or benefit.

E The executor may pay the interest to the son only if he is able to demonstrate a need for this money.

A

The executor is required to pay all interest to the son now, as he has reached the age of 18.

(B) The executor is required to pay all interest to the son now, as he has reached the age of 18. This situation concerns the executor’s power to maintain a minor. When property is held for a minor beneficiary, PRs may apply the income for the maintenance, education, or benefit of the minor. Otherwise, the PRs must accumulate the income. Once the beneficiary turns 18, ongoing income must be paid to them until their contingent interest is satisfied. Here, now that the son is 18, he is entitled to receive all future interest received on the sum held. When he turns 21, he will receive the capital plus any accumulated income. (A) is therefore incorrect. (C) is incorrect because the executor must pay the interest to the son now that he is 18. (D) is incorrect because the executor must give the interest to the son regardless of the purpose for which he plans to use the income. (E) is incorrect because the son does not have to demonstrate a need for the money.

How well did you know this?
1
Not at all
2
3
4
5
Perfectly
39
Q

A man who was not domiciled in the UK died, leaving his estate of £140,000 to his sister. The estate includes a house worth £120,000 in the UK and a bank account with a UK bank worth £20,000. The man made a lifetime gift of £10,000 from his UK bank account to his son two years ago.

Which of the following statements is correct with respect to the man’s estate?

A The estate is not an excepted estate for inheritance tax purposes because of the property in the UK.

B The estate is an excepted estate and does not have to file an inheritance tax account.

C The estate owes inheritance tax because the man died as a non-UK domiciliary.

D The estate is an excepted estate, and so the PRs have to deliver an account to HMRC within six months from the end of the month of the man’s death.

E The estate is not an excepted estate because of the gift to the son.

A

The estate is not an excepted estate for inheritance tax purposes because of the property in the UK.

(A) The estate is not an excepted estate because of the property in the UK. An estate which owes no inheritance tax (‘IHT’) and does not have to file a formal IHT account is called an excepted estate. To be excepted, the estate’s total gross value plus specified transfers cannot exceed either (1) the inheritance tax threshold, currently £325,000, or (2) £3 million if the net chargeable estate does not exceed the threshold. In addition, if the deceased was not domiciled in the UK, an estate can be excepted only if the deceased’s UK estate consists solely of cash and/or quoted shares which do not total more than £150,000. Here, the man owned a house worth £120,000 in the UK. Consequently, the estate is not excepted, and an IHT account is required. (B) is, therefore, incorrect. (C) is incorrect. The man being a non-UK domiciliary does not mean his estate owes inheritance tax. Even if he was not domiciled in the UK, the deceased’s estate will still owe inheritance tax on the UK estate. (D) is incorrect because the estate is not excepted. Also, an IHT account is needed, but it must be delivered to HMRC within 12 months (rather than six months) from the end of the month of the man’s death. (E) is incorrect. The gift of £10,000 to the son is considered a specified transfer that must be added to the value of the estate. An estate can be excepted if specified transfers do not exceed £250,000 in the seven years before the deceased’s death

How well did you know this?
1
Not at all
2
3
4
5
Perfectly
40
Q

A defendant is standing trial for burglary. He is jointly charged with his nephew. The nephew pleads guilty before the start of the trial. Both the prosecution and the defendant want the nephew to give evidence. The nephew is tired of dealing with the criminal justice system and does not want to give evidence.

Which of the following best describes whether the defendant’s nephew will have to give evidence?

A The nephew is competent and compellable for both the prosecution and defence.

B The nephew is competent for both the prosecution and defence but only compellable for the defence.

C The nephew is neither competent nor compellable for the prosecution but both competent and compellable for the defence.

D The nephew is neither competent nor compellable for the prosecution but competent but not compellable for the defence.

E The nephew is competent but not compellable for both the prosecution and the defence.

A

The nephew is competent and compellable for both the prosecution and defence.

(A) The nephew is competent and compellable for both the prosecution and defence. Co-defendants are not competent or compellable for the prosecution, but they are competent but not compellable for the defence. However, if a co-defendant pleads guilty or the case is dropped, the co-defendant becomes an ordinary witness. Ordinary witnesses are competent and compellable for both parties. Here, after pleading guilty, the nephew became an ordinary witness and is both competent and compellable for the prosecution and the defence. This means either side can force him to give evidence. (B), (C), (D), and (E) are incorrect as they do not identify that the nephew is both competent and compellable for either side.

How well did you know this?
1
Not at all
2
3
4
5
Perfectly
41
Q

A buyer and a seller of a property are ready to exchange contracts. Their solicitors have agreed to use the Contract Incorporating the Standard Conditions of Sale (5th edition - 2018 revision), unamended, and the Law Society Formulae for exchanging contracts by telephone. The solicitors both have time available the following week and wish to exchange using Formula B.

How will the solicitors effect exchange of contracts?

A The buyer’s solicitor will send the contract signed by the buyer to the seller’s solicitor, along with the deposit. The seller’s solicitor will confirm both contracts are the same over the telephone, insert the exchange and completion dates in both, and send the part signed by the seller to the buyer’s solicitor.

B Each solicitor will hold their own client’s contract and will confirm the versions are the same over the telephone. They will then send their client’s signed part to each other, and the buyer’s solicitor will send the agreed deposit to the seller’s solicitor to hold as stakeholder.

C The buyer’s solicitor will send the contract signed by the buyer to the seller’s solicitor, who will confirm both contracts are the same, insert the exchange and completion dates in both, and send the part signed by the seller to the buyer’s solicitor. The buyer’s solicitor will then hold the agreed deposit as stakeholder.

D Each solicitor will hold their own client’s contract and will confirm the versions are the same over the telephone. They will then send their client’s signed part to each other, and the buyer’s solicitor will hold the agreed deposit funds as stakeholder.

E Each solicitor will send the other their client’s signed contract. The solicitors will confirm the versions are the same over the telephone, and the buyer’s solicitor will then send the deposit funds.

A

Each solicitor will hold their own client’s contract and will confirm the versions are the same over the telephone. They will then send their client’s signed part to each other, and the buyer’s solicitor will send the agreed deposit to the seller’s solicitor to hold as stakeholder.

the contract, confirms they are the same, and then sends it. The buyer’s solicitor will also send the deposit funds to the seller’s solicitor to hold as stakeholder pursuant to the unamended Standard Conditions of Sale. (A) is incorrect, as it describes a Formula A exchange – the buyer’s solicitor will send the contract signed by the buyer to the seller’s solicitor, along with the deposit. The seller’s solicitor will confirm both contracts are the same over the telephone, insert the exchange date in both, and send the part signed by the seller to the buyer’s solicitor. (C) is incorrect because it describes a Formula A exchange, and it also incorrectly provides that the buyer’s solicitor will hold the deposit. (D) is incorrect because although it describes a Formula B exchange, as just explained under the unamended Standard Conditions of Sale, the seller’s solicitor will hold the deposit funds as stakeholder. (E) is incorrect because no formula for exchange requires both solicitors to send their client’s contract to the other solicitor before confirming they are the same

How well did you know this?
1
Not at all
2
3
4
5
Perfectly
42
Q

In her will a woman appointed two trustees to hold her residuary estate on trust for her son for life with remainder to his children. The trust fund includes a plot of land which the trustees decide to sell. The land has been valued at £100,000. After obtaining informed consent from the woman’s son and the other trustee, one of the trustees buys the land from the trust at a public auction for £100,000.

Did the trustee act properly in purchasing the land?

A Yes, because he obtained the informed consent of the son.

B Yes, because he appears to have paid fair value for the land.

C No, due to potential conflicts of interest.

D Yes, because the land was sold in a public auction.

E Yes, because he obtained the consent of his co-trustee.

A

No, due to potential conflicts of interest.

(C) The trustee may not purchase the land due to potential conflicts of interest. Trustees must not place themselves in a position where their personal interests conflict with their fiduciary duties to the trust. The ‘self-dealing rule’ applies to prevent a trustee from purchasing trust property, and any such purchase is voidable at the instance of the beneficiaries. (A) is incorrect because a trustee’s purchase of trust property remains voidable, even if a beneficiary’s consent was obtained. (B) and (D) are incorrect because the rule applies to whatever sum the trustee pays for the trust property and regardless of whether it is sold at auction. (E) is incorrect because the consent of a co-trustee does not prevent the application of the self-dealing rule

How well did you know this?
1
Not at all
2
3
4
5
Perfectly
43
Q

A landlord owns the freehold of a unit which it let to the original tenant under the terms of a lease dated 20 July 2009. The original tenant assigned his interest in the lease to the second tenant 10 years ago. The second tenant assigned his interest in the lease to the current tenant five years ago and provided the landlord an Authorised Guarantee Agreement. The lease still has eight years left to run in its term. The current tenant is in arrears with his rent and is on the verge of bankruptcy.

Which of the following best describes the steps that the landlord could take regarding the rent arrears?

A The landlord must sue the current tenant for the rent arrears before the current tenant’s bankruptcy is finalised.

B The landlord must wait until the bankruptcy order is made against the current tenant and then contact the Trustee in Bankruptcy to demand payment as a creditor.

C The landlord may sue the current tenant or the second tenant for the rent arrears.

D The landlord must bring proceedings against the second tenant for forfeiture of the lease in order to collect any rent arrears.

E The landlord can bring proceedings against the original tenant or the current tenant for non-payment of rent.

A

The landlord may sue the current tenant or the second tenant for the rent arrears.

(C) The landlord may sue the current tenant or the second tenant for the rent arrears. In leases made after 1 January 1996, tenants are automatically released from their covenants upon assignment. A tenant will not be liable for a subsequent tenant’s breach of covenant after the assignment. However, as a condition of giving consent to an assignment, a landlord can require an outgoing tenant to enter into an Authorised Guarantee Agreement in which the outgoing tenant will act as guarantor for his immediate successor in title. In such cases, the landlord can sue the tenant-guarantor for the current tenant’s breach as well. Here, the lease is dated after 1 January 1996 and the second tenant guaranteed the current tenant’s performance. Thus, the landlord may pursue either the current tenant or the second tenant for the rent arrears. (A) and (B) are incorrect because, as explained above, the landlord does not need to proceed only against the current tenant. The landlord may seek to recover the rent arrears from either the second tenant due to the Authorised Guarantee Agreement or the current tenant (regardless of that tenant’s pending bankruptcy, though this situation may make collection difficult). (D) is incorrect because the landlord does not need to rely on forfeiture as the only means to collect the rent arrears, nor must they proceed only against the second tenant. As explained above, the landlord can sue either the current tenant or the second tenant for non-payment of rent. (E) is incorrect because the landlord cannot proceed against the original tenant. The original tenant was released from liability on assignment and provided no Authorised Guarantee Agreement under which he may have continuing liability.

How well did you know this?
1
Not at all
2
3
4
5
Perfectly
44
Q

A road barrier is erected at the end of a private road to stop members of the public accessing it. A man, angered by the restriction to access, unscrews the different sections to the road barrier and leaves them and the screws by the side of the road. The owner of the barrier hires a technician to put the barrier back in place. The technician is able to do so using all the original fittings, which are all still in perfect working order.

Is the man guilty of criminal damage?

A No, as none of the parts were damaged.

B No, as the technician was able to reassemble the barrier and it is in working order again.

C Yes, as the man was reckless about whether any damage occurred to the constituent parts of the barrier.

D Yes, because the impairment to usefulness was more than minimal.

E No, because the impairment to usefulness was minimal.

A

Yes, because the impairment to usefulness was more than minimal.

(D) The man is guilty of criminal damage. A person commits criminal damage when they destroy or damage property belonging to another, intending to do so or being reckless as to whether the property is damaged. Damage is anything that is more than minimal impairment of usefulness or value. Here, a technician is required to repair the barrier and put it back together again, so the damage is more than minimal. (A) and (B) are incorrect, as it is irrelevant that the barrier can be repaired and the parts are not damaged; the barrier’s usefulness was impaired. (C) is incorrect because the damage done here was intentional; the man deliberately dismantled the barrier. (E) is incorrect as the impairment to the barrier’s usefulness is more than minimal – it does not function at all

How well did you know this?
1
Not at all
2
3
4
5
Perfectly
45
Q

A woman would like to purchase a house on a large plot of land. She intends to use the space in the rear of the property for a garden featuring her prizewinning roses. The woman has heard, though, that one of the property’s neighbours has a right-of-way over the land in question which would prevent her from planting her garden. The woman’s solicitor has sent her a copy of the register of title of the land.

Where should reference to the right-of-way appear on the title?

A The Proprietorship Register.

B The Property Register.

C The Charges Register.

D The deed of easement which created the right-of-way.

E The conveyance which created the right-of-way.

A

The Charges Register..

(C) The Charges Register. The woman is interested in purchasing land which is subject to the burden of a right-of-way because another property enjoys an easement over that land. The burden of a right-of-way would appear on the Charges Register of the burdened title. (A) is incorrect as the Proprietorship Register specifies the class of title held and the name of the current holder or holders of the legal estate. (B) is incorrect as the Property Register identifies the property by its postal address and reference to the title plan. It also specifies the legal estate held, and, if a leasehold, it will give brief details of the lease (date, parties, term, rent, et cetera). It also will indicate whether the property has any benefits (such as rights-of-way) over neighbouring land. (D) and (E) are incorrect because the facts ask specifically where the reference should appear on the title – and the question makes it clear that this is a registered title. It might be necessary to look at a conveyance or deed if the right-of-way is not yet registered.

How well did you know this?
1
Not at all
2
3
4
5
Perfectly
46
Q

A woman owned the following shares:

5% of the shares in X Ltd which she bought for £1,000 5 years ago. X Ltd manufactures and sells ladies fashion items.
20% of the voting shares in Y PLC. Y PLC is not listed. It makes and sells kites. The woman inherited the shares from her father 8 years ago.
75% of the voting shares in Z PLC. Z is listed on the London Stock Exchange. Z builds high value flats in city centres and rents them out on short term lets. The woman bought the shares for £50,000 10 years ago.
The woman died last month leaving a valid will including the following gifts:

I give all my shares in X Ltd to my son, Marco.
I give all my shares in Y PLC to my best friend, Alice.
I give all my shares in Z PLC to my colleague, Simon.
All of the beneficiaries are still alive.

Which of the following best describes the legal position regarding the availability of business relief for the gifts made in the woman’s will?

A All 3 gifts will attract 100% business relief.

B The gift of shares in X Ltd will attract business relief at 100%, and the gift of shares in Y PLC and Z PLC will attract business relief at 50%.

C The gift of shares in X Ltd and Y PLC will qualify for 50% business relief, and the gift of shares in Z PLC will not attract business relief.

D The gift of shares in X Ltd and Y PLC and will qualify for 100% business relief, and the gift of shares in Z PLC will qualify for 50% relief.

E The gift of shares in X Ltd and Y PLC will qualify for 100% business relief, and the gift of shares in Z PLC will not attract business relief.

A

The gift of shares in X Ltd and Y PLC will qualify for 100% business relief, and the gift of shares in Z PLC will not attract business relief.

(E) Shares in an unlisted trading company qualify for 100% relief so long as the shares have been owned for at least 2 years. This rule applies regardless of the percentage of the shares held and regardless of whether the company is private or public. Here, the woman owned her shares of X Ltd for 5 years and Y PLC for 8 years. Ltd indicates the X is private, and a private company cannot be publicly traded. Therefore, X cannot be a listed company. A PLC is publicly traded and can be a listed company, but the facts indicate that Y is not a listed company. Therefore, the gifts of X shares and Y shares each attract the 100% deduction. Shares in a quoted trading company can attract a 50% relief, but only if the deceased had voting control over the company (that is, owned more than 50% of the company’s shares). Here, the woman owned 75% of the voting shares of Z PLC, and the facts indicate that Z PLC is a quoted company. However, the facts indicate that Z PLC builds flats and rents them out. Renting flats out is not a trading activity, and so the gift of Z shares does not qualify for business relief. (A) is incorrect because it indicates that the gift of Z shares would qualify for 100% relief. (B) is incorrect both because it indicates that the gift of Y shares would qualify for only 50% relief and because it indicates that the gift of Z shares would also qualify for 50% relief. (C) is incorrect because it indicates that the gifts of the X shares and Y shares would qualify for only 50% relief. And (D) is incorrect because it indicates that the gift of Z shares would qualify for 50% relief.

How well did you know this?
1
Not at all
2
3
4
5
Perfectly
47
Q

A man who was blind died recently. He made a will two years ago, leaving all of his assets to a friend. A neighbour has told the man’s children that the friend exerted a lot of influence over the man and behaved in a controlling manner towards the man in the final years of his life. The man’s children are upset not to receive anything under his will.

Which of the following best describes whether the children can challenge the validity of the man’s will?

A The children can challenge the validity of the will if they can prove that the friend’s actions amounted to coercion.

B The children have no basis on which to challenge the validity of the will.

C The children can challenge the validity of the will on the basis of lack of capacity.

D The children can challenge the validity of the will if they can prove that the actions of the friend amounted to strong persuasion.

E The children can challenge the validity of the will if they can rebut the presumption that the man acted with knowledge and approval when making the will.

A

The children can challenge the validity of the will if they can prove that the friend’s actions amounted to coercion.

(A) The children can challenge the validity of the will if they can prove that the friend’s actions amounted to coercion. For a will to be valid, the testator must have acted with an intention to make the particular will. A challenger may prove lack of intention by showing the testator made the will as a result of duress (that is, as a consequence of a threat of force), due to fraud, or because of undue influence (meaning coercion or pressure overpowered the testator’s free will). The children can, therefore, challenge the will on the basis of undue influence if they can prove that there was coercion or pressure that overpowered the freedom of action of the man. (B) is incorrect, as it may be possible to challenge the will on the basis of undue influence. (C) is incorrect. A will may be challenged for lack of capacity (that is, that the testator was unable to make decisions for themselves because of impairments of the brain). However, there is nothing to suggest that the man lacked capacity, and blindness does not affect a testator’s legal capacity. (D) is incorrect. Strong persuasion does not constitute undue influence. A higher level of pressure or coercion is required. (E) is incorrect. The usual presumption of knowledge and approval does not apply here, as the man was blind. This will assist the children in challenging the validity of the will

How well did you know this?
1
Not at all
2
3
4
5
Perfectly
48
Q

A settlor appoints two trustees to hold funds on trust for her grandchildren, all under the age of 18. The trust deed contains no express powers dealing with the appointment of trustees. One of the trustees is undergoing medical treatment, and she appoints an attorney to act on her behalf for six months. During this period the trustee dies.

Who has the power to appoint a replacement trustee?

A The surviving trustee.

B The settlor.

C The deceased trustee’s attorney.

D The deceased trustee’s personal representatives.

E The beneficiaries.

A

The surviving trustee.

(A) The surviving trustee has the power to appoint a replacement trustee. In the absence of express powers in the trust instrument, statutory provisions apply. Where a trustee dies, the surviving trustee has power to appoint a replacement. (B) is incorrect because there are no express powers in the trust instrument, so the settlor has no further power to appoint new trustees once the trust has been created. (C) is incorrect because the powers of an attorney come to an end when the principal dies. (D) is incorrect because a deceased trustee’s personal representatives only have power to appoint new trustees when a sole trustee dies. (E) is incorrect because the beneficiaries do not have power to appoint new trustees. (If all the beneficiaries are of full age and capacity and together absolutely entitled, they may direct the trustees to retire and appoint new trustees of their choice, but this is not the case here.)

49
Q

A settlor created a valid trust to benefit her adult child for life with the remainder to her grandchildren. The settlor appointed three trustees, one of whom the beneficiaries have never liked. The settlor has recently died, and the beneficiaries would like to remove the trustee from office. The settlor’s child is currently 58, and the grandchildren are aged 27 and 31. There is no person nominated in the trust instrument to appoint new trustees.

Can the beneficiaries remove the trustee from office?

A Yes, because all the beneficiaries agree that the trustee should be removed.

B Yes, because trustees are obligated to keep the beneficiaries happy.

C No, because beneficiaries have no power to remove or appoint trustees.

D No, because trustees can be removed by the court only.

E No, because the trustee was appointed by the settlor.

A

Yes, because all the beneficiaries agree that the trustee should be removed.

50
Q

A man drinks a bottle of whiskey whilst watching a football match and becomes extremely intoxicated. On his way home, he picks up a bottle from the street. He approaches a passerby and smashes the bottle on their arm, causing a deep gash. Police attend the scene. Upon arrest, the man punches the police officer in the face, giving him a black eye. The man is charged with s18 grievous bodily harm (‘GBH’) for the attack on the passerby and actual bodily harm (‘ABH’) for the attack on the police officer.

Which best describes the effect of intoxication on both these offences?

A ABH and s18 GBH are both basic intent offences, and so intoxication will not negate mens rea.

B ABH is a specific intent offence so intoxication can negate mens rea, but s18 GBH is a basic intent offence so intoxication will not negate mens rea.

C ABH is a basic intent offence so intoxication can negate mens rea, but s18 GBH is a specific intent offence so intoxication will not negate mens rea.

D ABH and s18 GBH are both specific intent offences, and so intoxication can negate mens rea.

E ABH is a basic intent offence and so intoxication will not negate mens rea, but s18 GBH is a specific intent offence and so intoxication can negate mens rea.

A

ABH is a basic intent offence and so intoxication will not negate mens rea, but s18 GBH is a specific intent offence and so intoxication can negate mens rea.

(E) Intoxication will not negate the mens area of ABH, but it can negate the mens rea of s18 GBH. Voluntary intoxication cannot negate the mens rea of basic intent offences, but it can negate the mens rea of specific intent offences. Offences of basic intent are those which can be committed either intentionally or recklessly. Specific intent offences cannot be committed recklessly; intention is required for the offence to be committed. ABH can be committed either by assault or battery, and so ABH’s mens rea is that of either assault or battery. The mens rea of assault is intention or recklessness regarding causing apprehension of immediate, unlawful violence. The mens rea of battery is intention or recklessness regarding the application of force. This means both forms of ABH can be committed recklessly, and so both are basic intent offences. By contrast, the mens rea of s18 GBH is intention to cause serious harm; the offence cannot be committed recklessly. This means s18 GBH is an offence of specific intent. For these reasons, (A), (B), (C), and (D) are incorrect.

51
Q

A settlor transfers land to trustees to hold on trust for his grandchildren. He prepares a written confirmation of the terms of the trust.

Who must sign the written confirmation in order to create a valid trust of the land?

A The trustees only.

B The settlor only.

C The trustees and the settlor.

D The settlor and the beneficiaries.

E The settlor or his lawfully authorised agent.

A

The settlor only.

(B) A declaration of trust of land must be evidenced by some writing signed by the settlor. (A), (C), and (D) are incorrect because there is no requirement for the declaration to be signed by the trustees or the beneficiaries. (E) is incorrect because the statute does not permit signature by an agent on the settlor’s behalf

52
Q

On 1 January 2023, a shopkeeper purchased a new commercial building in Wales for £600,000 plus £150,000 VAT. The stamp duty land tax rates for commercial buildings at the time were as follows:

  • 0% up to £150,000
  • 2% £150,001 - £250,000
  • 5% of amounts over £250,000

The Land Transaction Tax rates at the time were:

  • 0% up to and including £225,000
  • 1% £225,001 - £250,000
  • 5% £251,001 - £1 million
  • 6% on the portion over £1 million

What is the tax payable by the shopkeeper?

A £25,250

B £0

C £27,000

D £19,500

E £24,500

A

£25,250.

(A) As the property was in Wales then Land Transaction Tax (‘LTT’) is due rather than Stamp Duty Land Tax. LTT on a new commercial building is payable on the VAT inclusive price at the non-residential rates. Thus, the tax will be based on £750,000. Under the rates given, nothing is owed for the first £225,000. £250 is owed on the next £25,000 (1% of the amount between £225,000-£250,000) and £25,000 would be owed on the £500,000 purchase price above £250,000 (£500,000 x 5% = £25,000). £250 + £25,000 = £25,250.

53
Q

A man and his son buy a house, contributing equally to the purchase price. The house is conveyed into the name of the son alone to enable his father to claim state benefits by concealing his assets, which the father has done. The father and son are now estranged, and the father wishes to claim a share in the house. There is no written evidence to support his claim.

What is the likely result of the father’s claim?

A It will be up to the court to decide based on public interest.

B It will succeed because the presumption of resulting trust applies.

C It will fail because a trust of land requires signed written evidence to enforce it.

D It will succeed because the father does not need to plead his improper intention.

E It will fail because only the registered owner is entitled to land.

A

It will be up to the court to decide based on public interest.

(A) If a property transfer was made as part of an illegal or fraudulent transaction, the court must decide whether it is in the public interest to allow a claim. The court would take into account all relevant factors, including the underlying purpose of the relevant law and the respective conduct of the parties. In these circumstances, the court would likely consider whether creditors have in fact been deceived, whether the son was aware of the scheme, and the effect on either party of allowing the man’s claim. None of the other answer choices is as good as (A) because they each provide a definitive result. (C) is incorrect because the requirement that a declaration of trust of land be evidenced in signed writing does not apply to an implied trust, as would arise here if the court decides that is in the public interest. (E) is incorrect because, although the registered owner is entitled at common law, a claimant can establish an equitable interest under a trust.

54
Q

A testator died leaving a will in which he left the residue of his estate to trustees on trust for his widow for life with remainder to his nephew, provided he attains the age of 25. The will contains no express powers. The widow is still alive, and the nephew is 17 and about to go to university. He asks the trustees to give him some of the income to help with his university fees.

May the trustees use the income in this way?

A No, because the nephew is under 18 and cannot give them a valid receipt.

B No, because the nephew has only a contingent interest.

C No, because the will contains no express powers.

D No, because the trustees only have power to apply the income for the nephew’s maintenance.

E No, because the nephew has no interest in the income.

A

No, because the nephew has no interest in the income.

(E) The trustees cannot use the income to pay the nephew’s university fees because he has no interest in the income. The power to pay or apply income for a beneficiary’s maintenance, education, or benefit only applies where the beneficiary has an interest in the income. In this case, the income has been expressly given in the will to the testator’s widow, and so the trustees must pay the income to the widow and have no power to use it for the benefit of the nephew. (A) is incorrect because, although a child under 18 cannot give a valid receipt, this is not the reason why the trustees cannot pay the income to the nephew. (B) is incorrect because the power to use income for a beneficiary’s maintenance, education, or benefit does apply to beneficiaries with a contingent interest, but they also must have an interest in the income. (C) is incorrect because no express power would change the duty of trustees to pay the income to the beneficiary entitled to income under the will. (D) is incorrect because the nephew has no interest in the income. If he did, the trustees could use the income for the beneficiary’s education or benefit, not just for his maintenance.

55
Q

Question
A solicitor’s client is selling their property. The solicitor received a deposit, as a stakeholder, in the form of a cheque for £20,000 from the firm representing the buyer in this transaction.

Which of the following statements is correct with respect to the cheque?

A The solicitor should place the deposit in the client’s account, labelled as stakeholder money held for client and buyer.

B The solicitor must post the cheque directly to the client.

C The solicitor should immediately place the deposit in the business account for this client.

D The solicitor must place the deposit in the client account and hold it for the client alone.

E The solicitor must open a separate stakeholder account in the name of both buyer and seller.

A

The solicitor must open a separate stakeholder account in the name of both buyer and seller.

(E) A solicitor must record for whom they hold client money. Thus, the solicitor must open a separate stakeholder account in the name of both buyer and seller. (A) is incorrect; it was allowed under the previous version of the SRA Accounts Rules, but it is not allowed under the current version of the Rules. (B) and (D) are incorrect as until completion, the money does not only belong to the client alone – it belongs to both the client and the buyer. (C) is incorrect, as this is client money and not business money because it is not money available for the use of the solicitor.

56
Q

In December 1999, a man purchased a house for £176,000. The man has always rented out the house to tenants. In December 2000, he installed a new bathroom at a cost of £6,400. In January 2023, he sold the house for £642,000. He also paid stamp duty land tax at 1% of the purchase price when he bought the house.

What is the chargeable gain on disposal of the house?

A £466,000

B £459,600

C £445,540

D £457,840

E £464,240

A

(D) £457,840. On disposal of a chargeable asset, any purchase costs incurred at acquisition are deductible (for example, SDLT). The costs of any capital improvements are also deductible (for example, the new bathroom). The chargeable gain is always the gain pre-deduction of the annual exemption. So, it is sale price £642,000 – (£176,000 + £1,760 + £6,400) = £457,840

57
Q

A landowner grants a right of way to a grantee in a legal deed of easement. Title to the dominant and servient tenement is registered.

How will the right burdening the landowner’s land be reflected on his register of title?

A It will appear on the Charges Register.

B It does not need to be registered as it does not benefit the landowner’s land.

C It will appear on the Property Register.

D It will appear as a restriction on the Proprietorship Register.

E It will appear as a restriction on the Charges Register.

A

It will appear on the Charges Register.

(A) Legal easements are ones created by deed. The burden of a legal easement will appear as a notice on the Charges Register of the servient tenement (that is, the land burdened by the easement). Here, the landowner owns the servient tenement (the land burdened by the easement), so the burden will appear on the landowner’s Charges Register. (B) is incorrect because the right must be registered as a burden against the servient tenement. (C) is incorrect because a right benefitting land appears on the Property Register. The benefit of the easement will appear as a notice on the Property Register of the grantee’s register of title. (D) is incorrect because a restriction on the Proprietorship Register is not appropriate here. The Proprietorship Register indicates the current legal owners of the property. (E) is incorrect because while the burden will appear on the Charges Register, it will appear as a notice, not a restriction. A notice is an entry in respect of the burden of an interest affecting a registered estate or charge. A restriction is used to prevent any dealing with the land otherwise than in accordance with the terms of that restriction.

57
Q

A woman died last month, owning a house, cash of £30,000, and shares in HSBC plc and M&S plc. When the woman executed her will, she owned the house, shares in BT plc, and a Mercedes car.

Her will included the following provisions:

(1) I give my shares to my daughter.

(2) I give the remainder of my estate to my son.

There are no other relevant clauses.

Which of the following best describes entitlement to the woman’s estate?

A The son will inherit the entire estate.

B The daughter will inherit the shares in HSBC plc and M&S plc, and the son will inherit the remainder of the estate.

C The son will inherit the whole estate apart from the cash, as the woman did not have this money when she executed the will.

D The daughter is entitled to ask the personal representatives to buy shares in BT plc for her, and the son will inherit the remainder of the estate.

E The will is invalid, as the woman died owning different assets to those that she owned when she executed the will.

A

The son will inherit the entire estate.

(A) The son will inherit the entire estate. The basic rule of succession is that a will speaks from the date of death unless a contrary intention is apparent in the will. Therefore, the will disposes of the assets owned by the woman at the time when she died, even though these are different from the assets that she owned when she executed the will. However, this basic rule will not apply if a contrary intention is apparent in the will. When a testator makes a gift of “my shares”, they are normally taken to have shown an intention contrary to the basic rule. Instead, the testator intended for the beneficiary to receive the property as it was on the day of the will’s execution and not on the day of the testator’s death. Consequently, the gift of shares to the daughter will fail because the BT plc shares the woman owned when she wrote the will were not in her estate when she died. Therefore, the son will inherit the entire estate. (B) is incorrect. The gift of shares to the daughter will fail, and she will receive nothing. The son inherits the entire estate. (C) is incorrect. The will generally speaks from the date of death, so the cash will be covered by the gift of the residuary estate to the son even though the woman did not hold this cash when she executed the will. (D) is incorrect. The daughter is not entitled to ask the personal representatives to buy shares in BT plc for her. (E) is incorrect. Although particular gifts may fail, the will’s validity is not impacted by the changes in the woman’s assets after she made the will.

57
Q

A woman died intestate last week. The woman was survived by a brother, half-sister, grandparents, and several aunts.

Who is entitled to share in the distribution of the woman’s estate?

A The woman’s brother only.The woman’s brother only.

B The woman’s brother and half-sister.

C The woman’s brother, half-sister, and grandparents.

D The woman’s grandparents only.

E The woman’s brother, half-sister, grandparents, and aunts.

A

The woman’s brother only.

(A) Only the woman’s brother is entitled to inherit the woman’s estate. When a person dies intestate without a spouse or civil partner, the strict order of entitlement under the intestacy rules applies to determine who will inherit the estate. The order of entitlement begins with issue, parents, siblings of the whole blood, and then siblings of the half blood. In this situation, the woman’s brother is entitled to all of her estate to the exclusion of all other relatives. The brother has priority over the woman’s half-sister, who would be classified as a sister of the half blood. Accordingly, (B) is incorrect. (C) and (D) are incorrect for that reason and also because grandparents are entitled to share in an estate only if there are no living issue, parents, or siblings of the whole or half blood. (E) is incorrect for the reasons explained above; also, aunts and uncles come after grandparents in the order of entitlement.

58
Q

A man is upset that his neighbour plays loud music late into the night. The man decides that as soon as the neighbour is gone, he will burn the neighbour’s house down. The man purchased a petrol can, filled it, and brought it home. One evening, the music was silent. The man assumed his neighbour was gone for the night, and so he retrieved the can and some matches and walked toward his neighbour’s house. He opened his neighbour’s gate and started pouring petrol on the house. Fortunately, the neighbour was home and came outside when he heard his gate open. He tackled the man and prevented him from lighting a match to burn the house.

Which of the following presents the earliest time at which the man could be found guilty of attempting to commit arson?

A When the man purchased the petrol because that was the first step in preparation of completing the offence.

B When the man walked through his neighbour’s gate because he had taken a substantial step toward completing the offence.

C When the man poured the petrol on the house, because that is more than merely preparatory towards completing the offence.

D When the man decided to burn the house down, because that is when he formed the intent to commit the arson.

E When the man walked toward the neighbour’s house with petrol and matches in hand, because that satisfied the circumstance element of the offence but not the result element.

A

When the man poured the petrol on the house, because that is more than merely preparatory towards completing the offence.

(C) The earliest the man could be convicted of attempt is when he began pouring petrol on his neighbour’s house. Attempt is defined as an act which is more than merely preparatory in the commission of the offence. The man went beyond mere preparation when he started pouring petrol on the neighbour’s house. Consequently, (A), (B), (D), and (E) are incorrect because doing a preparatory act or merely deciding to commit a criminal offence do not go far enough toward completion. (B) and (E) are also incorrect because they state tests not used in the UK

59
Q

A woman witnessed a violent robbery in the street. She provided a statement to the police shortly after the incident, detailing events. She subsequently suffered a stroke and was taken to hospital where she remains. The suspected robber is now on trial.

Will the prosecution be able to adduce the woman’s written statement at court?

A Yes, because the statement is not hearsay.

B Yes, under the rule of res gestae.

C Yes, because the woman is not available to attend.

D No, as the prosecution could secure the woman’s attendance via video link.

E No, because the statement is hearsay.

A

Yes, because the woman is not available to attend.

(C) The prosecution will be able to adduce the woman’s written statement because the woman is not available to attend. Hearsay is a statement made outside of court, which is adduced to prove the truth of its content. A written witness statement is therefore hearsay, and so (A) is incorrect. Hearsay can be admissible if the witness is not available. A witness is considered unavailable when they are dead, unfit owing to bodily or mental condition, or outside of the UK and it is not reasonably practicable to secure their attendance. Here, the woman having a stroke and remaining in hospital will satisfy the requirement for unavailability, and so (C) is correct. (B) is incorrect. Res gestae is also a category of admissible hearsay, and it refers to when a witness was so emotionally overpowered by the events at the time of making the statement that concoction can be disregarded (think of the content of a panicky 999 call, for example). Here, there is no suggestion of the witness being emotionally overpowered at the time of making the statement. (D) is incorrect as in these kind of circumstances where the witness is in ill health the written statement can be adduced. (E) is incorrect because the statement is hearsay.

60
Q

A trustee was appointed under a trust of concurrent interests with two equally entitled beneficiaries. The trustee removed £10,000 from the trust bank account and gave it to their cousin. The cousin remarked to the trustee that they were surprised at the gift as they thought the trustee had no money. The trustee merely winked in reply. The trustee is bankrupt, so a tracing claim has little or no value.

Which of the following best states the legal position regarding the cousin’s liability?

A The cousin took the funds free of the interest of the trust beneficiaries because the cousin had no notice of the trust.

B The cousin likely acted with conscious impropriety and should not retain the funds.

C The cousin likely had sufficient knowledge of the circumstances such that it would be unconscionable for the cousin to retain the funds.

D The cousin likely facilitated a breach of trust by being a dishonest accessory.

E The cousin likely would be considered an innocent volunteer recipient.

A

The cousin likely had sufficient knowledge of the circumstances such that it would be unconscionable for the cousin to retain the funds.

(C) The cousin likely had sufficient knowledge of the circumstances such that it would be unconscionable for the cousin to retain the funds. A knowing recipient is a person who receives money or property traceable to a breach of trust with knowledge of the breach. The person must have sufficient knowledge as to make it unconscionable for the recipient to retain the property. Unconscionability will be found if the recipient’s knowledge falls into one of the following categories: (1) actual knowledge, (2) wilfully closing one’s eyes to the obvious, (3) wilfully and recklessly failing to make such inquiries as an honest and reasonable person would make, (4) knowledge of circumstances which would indicate the facts to an honest and reasonable person, or (5) knowledge of circumstances which would put an honest and reasonable person on inquiry. Here, the cousin thought that the trustee had no money, and the trustee merely winked when the cousin mentioned this. The cousin should have known that there was something suspicious about the money, and he would likely be considered to have wilfully closed his eyes to the obvious or at least have had knowledge that would put a reasonable person on inquiry. (A) is incorrect because it is part of the standard for bona fide purchasers. A person who acquires title to trust property for value and without notice of the trust takes the property free of the equitable interests of the beneficiaries. Here, the cousin received trust property for free and had knowledge of suspicious circumstances. (B) and (D) are incorrect because the cousin does not meet the definition of a dishonest accessory. A dishonest accessory is a person who dishonestly facilitates a breach of trust. Dishonesty is described as unconscious impropriety. Here, the cousin did not facilitate the breach of trust; the trustee had already removed it from the trust bank account. (E) is incorrect because an innocent volunteer recipient comes into possession of trust property with no knowledge or suspicion that a breach of trust has occurred. As explained above, the cousin had suspicions about the source of the money.

61
Q

A street robbery takes place in a crowded town centre. A shopper is at the scene of the robbery and goes to a nearby police station to provide a statement of what he witnessed. He describes the perpetrator as having a black hooded top, brown hair, and blue eyes. The police officer taking the statement goes back to the scene of the robbery with the shopper to see if the suspect is still in the vicinity. The shopper points out a person nearby who has a black hooded top, brown hair, and blue eyes and confirms that they are the perpetrator of the robbery. The suspect is arrested and subsequently denies the offence in interview.

Should an identification procedure be held?

A Yes, as the shopper has purported to identify the perpetrator.

B Yes, as the suspect’s appearance matches with the first description.

C No, as the street identification constitutes identification by confrontation.

D No, as the street identification means that the victim has recently seen the suspect and so there is no benefit in carrying out any further procedure.

E No, as the street identification constitutes identification by group identification.

A

Yes, as the shopper has purported to identify the perpetrator.

(A) An identification procedure should be held because the shopper has purported to identify the perpetrator. Identification procedures must be held when: (1) a witness has identified or purported to identify a suspect, (2) a witness expresses the ability to identify a suspect, or (3) there is a reasonable chance of a witness being able to identify the suspect. Here, the shopper has purported to identify the suspect. The sighting of the suspect in the vicinity of where the offence took place is not a formal identification. As the suspect is not known to the witness, a formal identification process should now take place. (B) is wrong as it is irrelevant whether the first description matched the appearance of the suspect; the key is that the shopper has purported to identify the suspect. (C) and (E) are incorrect because confrontation and group identification are two types of formal (if unusual) identification procedures that can be used once a suspect has been identified. Here, the suspect was not known at the time of the street identification, so the street identification was neither type of identification procedure. (D) is wrong as the suspect is not known to the witness, and so an identification procedure should still be held.

62
Q

A property owner is concerned because his neighbour has decided to keep pigs on their land. The property owner believes this activity breaches a covenant entered into by the property owner’s predecessor in title. The covenant states that there must be ‘no noxious odorous or offensive activity’ taking place on the neighbour’s land.

Which of the following is a condition that must be satisfied for the property owner to enforce the benefit of the covenant at law?

A The property owner’s predecessor in title must have once held the legal estate in the land to be benefitted as well as the land to be burdened.

B The neighbour’s land must be part of a building scheme of development.

C The covenant must be positive in nature.

D The covenant must be personal in nature.

E The covenant must have been intended to run with the land held by the predecessor in title.

A

The covenant must have been intended to run with the land held by the predecessor in title.

(E) One condition the property owner must satisfy is that the covenant must have been intended to run with the land held by the property owner’s predecessor in title. Additionally, in order to enforce the benefit of the covenant at law, the property owner must show that (1) the covenant touches and concerns his land; (2) at the time the covenant was made, the property owner’s predecessor in title held the legal estate in the land to be benefitted; and (3) the property owner now holds the legal estate in the land to be benefitted. (A) is incorrect because the property owner’s predecessor in title did not need to have held both the legal estate in the land to be benefitted and the land to be burdened for the property owner to enforce the covenant. As explained above, the predecessor in title needed to have held only the legal estate in the land to be benefitted. (B) is incorrect because whether the neighbour’s land is part of a building scheme may be relevant to enforcement of the covenant in equity, but the question asks about enforcement at law. (C) is incorrect because in addition to not being a relevant condition, positive covenants generally bind only the original covenantor and are difficult to enforce. (D) is incorrect because the opposite is true: the covenant must not be personal in nature in order for the property owner to enforce it. In other words, the covenant must touch and concern the land to be benefitted.

63
Q

A couple are purchasing a property and note from the Property Information Form that the seller built a garage at the property eight years ago. The buyer’s local search reveals no planning permissions relating to this property.

Can the local authority take enforcement action against the buyers after the purchase completes?

A Yes, because the local authority has 10 years to take enforcement action for lack of planning permission.

B No, because the time limit for enforcement action for lack of planning permission is one year.

C Yes, because the local authority has unlimited jurisdiction to take enforcement action for lack of planning permission.

D No, because the time limit for enforcement action for lack of planning permission is four years.

E No, because the time limit for enforcement action for lack of planning permission is five years.

A

No, because the time limit for enforcement action for lack of planning permission is four years.

(D) The time limit for enforcement for unauthorised building work is four years, so the enforcement period passed four years ago, as the garage was built eight years ago. (A) is incorrect. A 10-year enforcement period applies to unauthorised material change of use and not to building without permission. (B) is incorrect as the one-year enforcement period applies to actions to enforce building regulations. (C) is incorrect as the unlimited jurisdiction time limit applies to when a local authority seeks an injunction to require an owner to bring a building up to the building regulations standards. (E) is incorrect because the period is four rather than five years

64
Q

A defendant is convicted at trial in the Magistrates’ Court of one charge of actual bodily harm (‘ABH’) and one charge of criminal damage, arising from the same facts. The magistrates proceed immediately to sentence.

What is the maximum sentence the magistrates could impose on the defendant?

A One month’s imprisonment.

B Three months’ imprisonment.

C Six months’ imprisonment.

D 12 months’ imprisonment.

E A fine.

A

(D) The maximum sentence the magistrates could impose on the defendant is 12 months’ imprisonment. Both ABH and criminal damage are either way offences. The Magistrates’ Court can impose a maximum sentence of 12 months’ imprisonment for two or more either way offences. For this reason, (A), (B), (C), and (E) are incorrect.

65
Q

A football supporter is at a football match. She has bought some crisps to snack on and is sitting next to an aisle in the stadium waiting for the match to begin. As she waits, she starts throwing her crisps on the floor in boredom, to see how far she can make them go down the aisle. She foresees the risk that she might hit someone with a crisp and it is an unreasonable risk to take, but she continues throwing the crisps. One of the crisps lands on another spectator.

Is the football supporter guilty of battery?

A Yes, because she had indirect intention.

B No, because she did not have direct intention.

C Yes, because she was reckless.

D No, because there was no direct application of force.

E Yes, because of transferred malice.

A

Yes, because she was reckless.

(C) The football supporter is guilty of battery. Battery is the intentional or reckless application of unlawful personal force upon another person. The test for recklessness is whether the defendant foresaw the risk of the harm occurring and, in the circumstances known to the defendant, it was an unreasonable risk to take. Here, the football supporter foresaw the risk that the crisp might hit someone. There is no social utility in throwing crisps, so it is an unreasonable risk to take in these circumstances. (A) is incorrect because indirect intention is only available to specific intent offences and battery is a basic intent offence. (B) is incorrect because recklessness can also satisfy the mens rea requirements for battery. (D) is incorrect as battery can be committed by indirect application of force. (E) is incorrect as transferred malice does not apply where the offence intended and the offence committed are different.

66
Q

Question
A woman sends a signed letter to her son in which she states that she gives him her cottage, Greenacre, to hold on trust for the son’s two daughters equally. The following week, the woman dies. Greenacre is still registered in her name. In her will she appoints her son to be her executor and leaves all her estate to a named charity.

A No, because it is incompletely constituted.

B Yes, because the son takes the legal title as executor.

C No, because there is no written declaration of trust.

D No, because there is no signed writing to prove its terms.

E Yes, because the every effort test is satisfied.

A

Yes, because the son takes the legal title as executor.

(B) The trust is valid because the son takes the legal title as executor. The woman was attempting to create a trust of land with her son as trustee and his daughters as beneficiaries. The requirements for a valid declaration of trust are that the three certainties and the beneficiary principle must be satisfied and, where the trust property is land, the declaration and its terms must be supported by evidence in signed writing. Here, the letter to the son demonstrates certainty of intention (the son is to hold the property on trust), certainty of subject matter (Greenacre), and certainty of objects (the son’s two daughters). The beneficiary principle is met as the two daughters are ascertainable human beneficiaries. In addition, the settlor must transfer the land to the trustee by means of a deed of transfer which must be registered at HM Land Registry. Where no such transfer is made, the trust is said to be incompletely constituted. Such a trust will usually fail unless one of the exceptions applies. The every effort test applies where the settlor has done everything required of her to complete the transfer of the legal title, and has put the property outside her control. The woman has not satisfied this test because she did not complete a deed transferring the legal title and deliver it to the trustee. A second exception is the rule in Strong v Bird which applies where the settlor makes an attempt to transfer the title which fails, her intention continues up to her death, and the intended trustee or donee acquires the legal title as her personal representative. All these requirements are met in this case. The letter was a failed attempt to transfer the legal title, and there is nothing to suggest that the settlor had changed her mind before she died. Therefore, the son takes the legal title as executor and holds Greenacre on trust for his daughters. (A) is incorrect because, although the trust is incompletely constituted, it does not fail. (C) is incorrect because there is no requirement for a declaration of trust to be in writing. (D) is incorrect because the signed letter is sufficient to prove the terms of the trust. (E) is incorrect because, as stated above, the every effort test is not satisfied.

67
Q

A woman sets fire to a caravan and watches it burn to the ground.

Which of the following would mean the woman was not guilty of the charge of aggravated arson?

A The caravan belonged to the woman.

B The woman had checked that there were no people inside or near the caravan before setting it on fire.

C The woman has the consent of the caravan’s owner to set it on fire.

D The woman did not intend to endanger life.

E No one was injured or killed as a result of the fire.

A

(B) The woman would not be guilty of aggravated arson if she had checked that there were no people inside or near the caravan.

Arson is committing criminal damage by fire. Aggravated arson has all the elements of ordinary arson, but there is an additional mens rea requirement that the defendant must intend or be reckless as to the endangerment of life by the damage caused to the property. If the woman had checked that there were no people in or near the caravan, she would not have intended or been reckless as to the endangerment of life. (A) is incorrect as the property damaged can belong to the defendant or to someone else for aggravated arson. (C) is incorrect as the ordinary defences applicable to criminal damage do not apply. (D) is incorrect as recklessness will suffice for aggravated arson. (E) is incorrect as there is no requirement for death or injury to result; the additional requirement relates to mens rea rather than actus reus

68
Q

A defendant appears in the Magistrates’ Court charged with battery. The defendant earns minimum wage in a meatpacking warehouse and has no savings. He is of good character and is likely to lose his job if convicted.

What are the defendant’s options for representation in court?

A A representation order can be granted if the interests of justice test is passed, or the defendant can privately fund his representation.

B A representation order can be granted if the means test is passed, or the defendant can privately fund his representation.

C A representation order can be granted if the interests of justice test is passed, but privately funding representation is not possible in the Magistrates’ Court.

D A representation order can be granted if the interests of justice test and the means test are passed, but privately funding representation is not possible in the Magistrates’ Court.

E A representation order can be granted if the interests of justice test and the means test are passed, or the defendant can privately fund his representation.

A

(E) A representation order can be granted if the interests of justice test and the means test are passed, or the defendant can privately fund his representation. (A) and (B) are incorrect as they include only part of the test for the grant of a representation order for publicly funded representation. The defendant must satisfy two requirements for an order to be awarded: the interests of justice test and the means test. Private funding is always an alternative to public funding, and so (C) and (D) are incorrect.

69
Q

A woman is selling her property and has found an interested buyer. For the last five years, the woman has allowed her neighbour to take a shortcut across her land to reach a nearby park. However, this arrangement has never been formalised. A worn path has now been created in the rear of the property by the neighbour’s regular crossing.

Does the woman have a duty to disclose this use in the contract with the buyer?

A No, because the use is a latent defect, and latent defects need not be disclosed.

B Yes, because otherwise the woman would be liable for nondisclosure.

C No, because the use is a patent defect and patent defects need not be disclosed.

D No, under the doctrine of ‘caveat emptor’.

E Yes, because the use is a patent defect and patent defects must be disclosed.

A

No, because the use is a patent defect and patent defects need not be disclosed.

(C) The woman need not disclose the use, as it is a patent defect. A right-of-way which is visible, such as the one in the question, is a patent defect. It does not need to be disclosed in the contract because the seller’s duty of disclosure does not extend to patent defects. (Note that a prudent solicitor would advise the woman to disclose the right-of-way anyway – to avoid any possible misrepresentation claim – but disclosure is not actually required under the circumstances.) (A) is incorrect, both because the defect is patent (obvious) rather than latent (not obvious), and because the rule is the opposite of that stated – latent defects must be disclosed. (B) is incorrect because for the reasons just explained – disclosure is not required here, so non-disclosure will not result in liability. (D) is incorrect as the doctrine of ‘caveat emptor’ (that is, let the buyer beware) is the reason that the defect does not need to be referred to in the contract – because the defect is observable, the doctrine of caveat emptor applies and the buyer has the burden of discovering the defect. (E) is incorrect because it has the rule backwards – patent defects need not be disclosed, as discussed above.

70
Q

A man and his brother died in an accident last month. The man’s valid will appointed his brother, sister, and 14-year-old son as his executors. The man’s wife and 18-year-old daughter are also alive.

Who has the best right to apply for a grant of representation to the man’s estate?

A The sister only.

B The sister and wife only.

C The sister and son only.

D The sister, wife, and daughter only.

E The wife only.

A

(A) The sister should apply for a grant of representation (specifically, a grant of probate), as she is the executor named in the will. The man’s brother is also named as an executor, but this appointment cannot take effect because he has died. Minors also cannot take a grant, so the 14-year-old son cannot apply. Therefore, (C) is incorrect. As the sister has the right to apply for the grant as an executor under the will, the other family members do not have the right to apply for a grant. If the man had died intestate or without appointing an executor in his will, other family members might be eligible to apply. Consequently, the man’s wife and daughter have no right to a grant, and (B), (D), and (E) are incorrect.

71
Q

A witness is giving evidence for the defence in a trial for theft. The witness has provided an alibi to the defendant, and she has a previous conviction for theft herself. In examination in chief, unprompted, the defence counsel asks, “You were at your home with the defendant at the time of the theft, weren’t you?”. This is a matter in issue between the prosecution and defence, as the prosecution maintains the defendant was at the shop where the theft occurred.

What recourse, if any, does the prosecution have?

A Because of the defence’s leading question, the prosecution are entitled to ask leading questions in cross-examination as well.

B The prosecution have no recourse, as the defence are entitled to ask leading questions during examination in chief.

C The prosecution are entitled to object to the question because it is a leading question.

D Because of the defence’s leading question, the prosecution can seek to undermine the reliability of the witness’s evidence.

E Because of the defence’s leading question, the prosecution can seek to undermine the integrity of the witness’s evidence by adducing her previous conviction.

A

The prosecution are entitled to object to the question because it is a leading question.

(C) The prosecution are entitled to object to the question because it is a leading question. Leading questions are not permissible during examination in chief, unless they are regarding issues which are not disputed. (A), (D), and (E) are incorrect, as whilst the prosecution can ask leading questions during cross-examination, seek to undermine the quality of the evidence, and adduce the witness’s bad character, none of these are related to the prosecution asking a leading question. (B) is incorrect as the prosecution does have recourse

72
Q

Question
A buyer completed the purchase of a piece of land with unregistered title last week.

When does legal title transfer to the buyer?

A Upon exchange of contracts.

B When the money passes from buyer to seller.

C Once the disposition is registered at His Majesty’s Land Registry.

D Upon completion.

E Once the agreement to buy and sell is made.

A

(D) The legal title in an unregistered transaction passes on legal completion (that is, the point in the transaction when the purchase price is paid by the buyer and the document of transfer is handed to the buyer’s solicitor). (A) and (E) are incorrect because exchange is the point that the agreement to buy and sell is created and the completion date is set. While equitable title passes on exchange, no legal title passes at this point in the transaction. (B) is incorrect because while a transfer of money may happen in conjunction with completion, the transfer alone is not determinative; the occurrence of legal completion is. (C) is incorrect because it describes when legal title passes in the registered system (that is, upon registration of the disposition at His Majesty’s Land Registry)

73
Q

In his will a man gave a legacy of £10,000 to his son “to hold on trust on such terms as I may communicate to him”, and he left the residue of his estate to his wife. Before he signed the will, the man explained to his son that he wished him to hand the £10,000 to a named friend and his son agreed.

Which of the following best describes the reason why the trust fails?

A Its terms are not set out in the will.

B The communication was not in writing.

C The communication was not consistent with the wording of the will.

D The friend was not named in the will.

E The man did not transfer the money to his son.

A

(C) The trust fails because the communication was not consistent with the wording of the will.

The usual rule is that a trust which is intended to take effect on death and is to be revocable until then must be declared in a valid will. However, where a will contains a declaration that a gift is to be held on trust but does not disclose the identity of the beneficiaries, a valid half-secret trust arises provided that the identity of the beneficiary is disclosed to the trustee before the date of the will, and that disclosure is consistent with the wording in the will. Here, the wording of the will (“on such terms as I may communicate to him”) points to a future communication and will be regarded as inconsistent with the actual communication which has already taken place when the will is made. (A) is incorrect because a half-secret trust may be valid even though its terms are not set out in the will. (B) is incorrect because there is no need for the communication to be in writing; all that is required is evidence that the communication took place. (D) is incorrect because a half-secret trust may be valid even though the beneficiary is not named in the will. (E) is incorrect because where a valid trust is created in a will it is the role of the executors to transfer the legal title to the trustees.

74
Q

Trustees are holding a fund on trust for a woman for life with remainder to her two sons in equal shares. The woman has just died, and the sons discover that 10 years ago the trustees paid the mother £50,000 from the capital of the fund to assist her to buy a house in the UK.

Do the sons have a viable claim against the trustees for breach of trust?

A No, because land in the UK is an authorised investment.

B Yes, because the trustees had no power to pay capital to the mother.

C No, because the breach of trust took place 10 years ago.

D Yes, because the purchase of a house as a residence is not an investment.

E No, because the trustees made proper use of their power to advance capital.

A

(B) The sons can bring a claim against the trustees for breach of trust because the trustees had no power to pay capital to the mother.

Trustees have power to advance capital for a beneficiary’s advancement or benefit provided that the beneficiary has an interest in capital. Here, the mother was a life tenant with an interest in the income only, and so the trustees were in breach of trust when they paid capital to her. (A) is incorrect because, although the purchase of land in the UK is an authorised investment, the trustees did not buy land for the trust. They handed trust funds to the mother. (C) is incorrect because for limitation purposes time does not run against a beneficiary under a trust until the beneficiary’s interest falls into possession, which happened only on the mother’s death. (D) is incorrect because the purchase of land as a residence is an authorised investment, and, in any event, these trustees were not buying land for the trust. (E) is incorrect because the trustees only have power to advance capital to a beneficiary with an interest in capital

75
Q

Question
A university student regularly looked at his phone while he walked to and from his classes. Although his friends did the same to a certain extent, the student was known for being an especially distracted walker. His friend was worried that someday the student might be hit by a car. To teach the student to pay more attention to his surroundings, and intending to frighten him, the friend jumped out from behind a light pole near a crosswalk waving a realistic-looking plastic knife. He shouted at the student, “Give me your phone!” The friend did not intend to use the plastic knife to physically harm the student; he intended only to show it to scare the student. The student was so frightened, he fell over awkwardly and fractured his arm in two places.

Which of the following is the most serious crime for which the friend could be convicted?

A Wounding or causing grievous bodily harm with intent.

B Wounding or causing grievous bodily harm without intent.

C Robbery.Robbery.

D Theft.Theft.

E Common assault.

A

(B) The most serious crime for which the friend could be convicted is wounding or causing grievous bodily harm without intent.

This crime requires causing grievous bodily harm with intention or recklessness regarding some harm. Here, the friend’s actions caused the student to fracture his arm. The friend did not intend for the student to break his arm. However, the test for recklessness is whether the defendant foresaw the risk of causing some harm and whether, in the circumstances known to the defendant, it was an unreasonable risk to take. Here, the friend must have foreseen the risk of causing some harm by jumping out and waving a realistic-looking knife, and it would certainly be an unreasonable risk to take. (A) is incorrect because wounding or causing grievous bodily harm with intent requires intent to cause grievous bodily harm, and the friend had no such intent. (C) and (D) are incorrect because the friend did not commit the required physical act or have the requisite intent for robbery or theft. For a person to be guilty of either of these two offences, he must dishonestly take a person’s property and intend to deprive the person of the property permanently. Neither requirement was satisfied here. The friend’s actions do constitute common assault, but wounding or causing grievous bodily harm is a more serious offence and so is a better choice than (E).

76
Q

The defendant’s friend is involved in a long-running dispute with his neighbour. The friend asks the defendant if he could borrow the defendant’s sledgehammer to smash his neighbour’s garden shed. The defendant does not approve of his friend’s plan, but he finds it difficult saying no to him. The defendant hands his sledgehammer to the friend, saying, “I think this is crazy, but here you go. I don’t want to be involved in this”. The friend leaves and returns an hour later, having caused serious damage to his neighbour’s shed. He returns the sledgehammer to the defendant.

Can the defendant be held criminally liable for the criminal damage to the shed?

A The defendant can be held liable for the criminal damage as an innocent agent because allowing a friend to borrow a sledgehammer is an innocent act.

B The defendant can be held liable for the criminal damage as a co-principal because he was involved in the crime at its inception.

C The defendant can be held liable for the criminal damage as an accomplice because he aided in the commission of the crime.

D The defendant cannot be held liable for the criminal damage because he said that he did not want to be involved in the crime.

E The defendant cannot be held liable for the criminal damage because he did not personally cause any of the damage.

A

(C) A person who aids, abets, counsels, or procures another offender is an accomplice and will also be guilty of the crime committed.

Here, the defendant aided his friend by allowing his friend to borrow the sledgehammer so that he could damage his neighbour’s shed. (A) is incorrect because it misstates what an innocent agent is. An innocent agent is not guilty of a crime. An innocent agent is one who unknowingly aids in an offence. Here, the defendant knew his sledgehammer was to be used in the commission of a crime and so was not an innocent agent. (B) is incorrect because a co-principal is one who acts together with another to commit the prohibited act, and here the defendant did not act with the friend in committing the act of criminal damage. (D) is incorrect because it is irrelevant; the fact that the defendant said he did not want to be involved does not negate his action of allowing his friend to use his sledgehammer to commit the crime. (E) is incorrect because the defendant does not have to personally commit the crime to be liable as an accomplice.

77
Q

A defendant is standing trial in the Crown Court on a charge of grievous bodily harm (‘GBH’). The defendant has a number of convictions for violent offences. As a result, the prosecution seek to adduce the defendant’s relevant previous convictions. The defence have made a counter application that the evidence should be excluded.

How will the court decide whether to grant the defence application to exclude the previous convictions?

A The court must exclude the convictions if their admission would have an adverse effect on the fairness of the proceedings.

B The court may exclude the convictions if it would be in the interests of justice.

C The court must exclude the convictions if it would be in the interests of justice.

D The court may exclude the convictions if it would be reasonable in all the circumstances.

E The court may exclude the convictions if their admission would have an adverse effect on the fairness of the proceedings.

A

(A) The court must exclude the convictions if their admission would have an adverse effect on the fairness of the proceedings.

When evidence is sought to be adduced under the bad character evidence gateways of ‘an important matter in issue between the prosecution and defence’ or ‘the defendant attacked another’s character’, the court must not admit the evidence if: (1) the defence make an application to exclude it, and (2) it appears to the court that the admission of the evidence would have such an adverse effect on the fairness of the proceedings that the court must not admit it. The gateway of ‘an important matter in issue between the prosecution and defence’ permits admission of bad character evidence on the basis of propensity to commit similar types of offences. Therefore, the defendant’s past convictions for violent offences fit within this gateway. (B), (D), and (E) are incorrect as they use ‘may’ instead of ‘must’. (B), (C), and (D) are further incorrect, as they misstate the test to be applied by the court.

78
Q

A person is standing on a bridge over a motorway, throwing stones at the passing cars. The person foresees the risk of damaging the cars he is aiming at but does not foresee the risk of death. He also does not intend to kill anyone. One of the stones hits a passing car, causing it to crash. The driver of the car dies at the scene.

Is the person guilty of unlawful act manslaughter?

A Yes, as the act was unlawful, and there was an obvious risk of death.

B Yes, as the act was unlawful, dangerous, and intentional and caused the death of the victim.

C No, as the death was not intentional.

D No, as the person did not foresee the risk of death.

E No, as the act was not unlawful.

A

Yes, as the act was unlawful, dangerous, and intentional and caused the death of the victim.

(B) The person is guilty of unlawful act manslaughter because the act of throwing stones at cars was unlawful, dangerous, and intentional and caused the death of the victim. Unlawful act manslaughter arises when a person kills another while committing a criminal offence that carries an objective risk to the victim. The requirements are that the act (1) was unlawful, (2) was deliberate, (3) carried the risk of some harm, and (4) caused the death of the victim. Here the underlying offence is criminal damage, as the person intentionally or recklessly damaged property belonging to another. The act of throwing stones at cars was dangerous by an objective standard and was intentional, and there is a causal link between the act and the death. The person is therefore guilty of unlawful act manslaughter. (A) is incorrect, as an obvious risk of death is not required for unlawful act manslaughter. (C) is incorrect, as the act must be intentional, not the death. (D) is incorrect, as the defendant need not foresee the risk of death. (E) is incorrect, as damaging property belong to another (here, the cars) whilst foreseeing the risk of damage is unlawful; it constitutes criminal damage.

79
Q

Two business partners purchased a freehold factory unit together in 2011 and contributed equally to the purchase price. They both appear on the title as registered proprietors and hold the beneficial interest as joint tenants. They have received some recent tax advice and the partners now wish to bring their joint tenancy to an end but still wish to remain co-owners of the property.

What steps must be taken to formalise the partners’ desired position?

A The joint tenancy cannot be severed once entered into so the partners cannot achieve their desired position.

B The joint tenancy in law must be severed and a form A restriction placed on the proprietorship register.

C The joint tenancy in equity must be severed and a form A restriction placed on the proprietorship register.

D The joint tenancy in equity must be severed and no further action is required.

E The joint tenancy in equity must be severed and a form A restriction placed on the property register.

A

The joint tenancy in equity must be severed and a form A restriction placed on the proprietorship register.

(C) To formalise the partners’ desired position, the joint tenancy in equity must be severed and a form A restriction placed on the proprietorship register. Co-owners may initially buy a property and hold as joint tenants at the outset, but their intentions may change and they may wish to convert their interests to a tenancy in common. It is possible to bring the joint tenancy in equity to an end and convert it to a tenancy in common through severance (including severance by mutual agreement of the joint tenants). Since the legal estate must be held as a joint tenancy, it is only possible to sever the joint tenancy in equity. When the beneficial interest is held as tenants in common, there will be a standard form A restriction in the proprietorship register that prevents dealings with the land other than in accordance with the terms of that restriction. Here, the partners have mutually agreed to bring their joint tenancy to an end, though they wish to remain co-owners. Thus, they will need to sever their joint tenancy in equity and place a form A restriction on the proprietorship register to indicate that the beneficial interest is now held by them as tenants in common. (A) is incorrect because, as explained above, a joint tenancy in equity can be brought to an end through severance, so the partners can achieve their desired position. (B) is incorrect because the joint tenancy in law cannot be severed – the legal estate must be held as a joint tenancy. (D) and (E) are incorrect because a form A restriction must be placed on the proprietorship register (not the property register) to indicate the partners’ beneficial ownership of the property as tenants in common.

80
Q

A man died on 8 December 2022, having made a lifetime cash gift of £500,000 to a trust on 16 October 2021. The man paid the inheritance tax arising from this gift.

Who will be responsible for paying the additional inheritance tax arising from the gift made to the trust as a result of the man’s death, and when will this be due?

A It will be paid by the executors of the estate and due on the 8 June 2023.

B It will be paid by the trustees and due on the 30 June 2023.

C It will be paid by the executors of the estate and due on the 16 October 2023.

D It will be paid by the trustees and due on the 8 June 2023.

E It will be paid by the executors and due on the 30 June 2023.

A

(B) The tax will be paid by the trustees and due on the 30 June 2023.

Inheritance tax payable on lifetime gifts as a result of death is always paid by the recipient of the gift. The extra tax is due six months after the end of the month of death.

81
Q

A defendant appears at the Crown Court charged with section 20 grievous bodily harm (‘s20 GBH’). The prosecution case is that the defendant attacked the victim with a broken bottle during a clash in a pub. The victim is Chinese, and the prosecution allege the incident was racially motivated. The defendant accepts that she caused serious harm to the victim. She alleges that she was waving a bottle around and accidentally smashed it when she banged it into the victim. She accepts she was reckless, as she foresaw the risk of some harm occurring and it was an unreasonable risk to take. She has offered to plead guilty on this basis, but the prosecution do not accept this basis of plea.

What is likely to happen next?

A The defendant will plead not guilty and the matter will proceed to trial.

B The judge will sentence on the basis of the prosecution facts.

C The judge will sentence on the basis of the defence facts.

D The judge will conduct a hearing to decide on which basis to sentence.

E The judge will decide on which basis to sentence without conducting a hearing.

A

The judge will conduct a hearing to decide on which basis to sentence.

(D) The judge will conduct a hearing to decide on which basis to sentence. If the prosecution do not accept the defendant’s basis of plea, the court will hold a Newton hearing to settle the disputed facts. A Newton hearing takes the form of a trial in which witnesses are called to give evidence. If the hearing is settled in the prosecution’s favour, the defendant will lose any credit for the guilty plea. If the hearing is settled in the defendant’s favour, the defendant will retain any credit for guilty plea. (A) is incorrect because the matter will not go to trial, as the defendant accepts her guilt. (B) and (C) are incorrect, as the judge will not automatically sentence on the basis of either the prosecution or the defence. (E) is incorrect as the judge will hold a hearing to decide upon which basis to sentence.

82
Q

A defendant is standing trial for criminal damage of his neighbour’s car in the Magistrates’ Court. The case is being heard by a panel of magistrates. During his evidence the defendant accuses his neighbour of having caused the damage himself, saying the neighbour has made false claims on insurance before. The prosecution now wants to adduce the defendant’s previous convictions for criminal damage.

Who will hear the prosecution’s application to adduce the defendant’s previous convictions?

A A different panel of lay magistrates.

B A District Judge.

C The lay panel’s legal advisor.

D The current panel of lay magistrates.

E No one will hear the application because the admission of bad character evidence cannot be considered on the day of trial.

A

The current panel of lay magistrates.

(D) The current panel of magistrates will hear the prosecution’s application. In the Magistrates’ Court, the lay panel or District Judge (‘DJ’) is the sole arbiter of matters of both fact and law. When matters of law, such as the admissibility of evidence, have to be decided during the course of a trial, they will be heard by the lay panel or DJ. If the panel or DJ rule evidence inadmissible, they will simply put the evidence from their minds for the remainder of the trial. For this reason, (A), (B), (C), and (E) are incorrect. A matter of law will be dealt with by the panel or DJ hearing the case

83
Q

A doctor prescribed an experimental medicine for a patient. The doctor did not inform the patient that the manufacturer had reported a small risk that the medicine caused severe delusions. After taking the medicine the first day, the patient began to believe that his next-door neighbour was spreading false rumours about him. While still under the influence of the medication, the patient grabbed a knife from the kitchen, went to his neighbour’s house, and rang the doorbell. When the neighbour answered the door, the patient plunged the knife into his neighbour’s chest, killing him instantly.

May the patient be convicted of murder?

A No, because the patient’s intoxication was with a non-dangerous substance.

B Yes, because the patient’s intoxication was voluntary.

C No, because the patient’s intoxication was involuntary.

D Yes, because the patient was able to form the required mental state for murder.

E Yes, because murder is a basic intent offence.

A

Yes, because the patient was able to form the required mental state for murder.

(D) Murder is the unlawful killing of another human being with the intention to kill or cause grievous bodily injury. Here, the patient took a knife to his neighbour’s house and stabbed him in the chest when he answered the door, killing him. The rules on intoxication will not permit avoidance of liability here. (A) is incorrect because whilst the patient has indeed consumed a substance deemed non-dangerous, intoxicated intent is still intent. (B) is factually incorrect. Voluntary intoxication arises when the defendant decides to drink or take illegal drugs; here the patient was taking a prescription which he did not know would cause delusions (because his doctor failed to tell him of the risk of delusions), which points to (C) involuntary intoxication. However, this defence will not work here. Involuntary intoxication is a defence only if it prevents the defendant from forming the required mental state. Here, the patient’s delusions made him believe his neighbour was spreading false rumours about him; as already discussed, nothing in the facts indicates the delusions prevented the patient from intending to cause grievous bodily harm or death. Neither do such circumstances otherwise justify a killing – that a person is spreading rumours does not give one the right to kill the person. Thus, the patient will still be criminally responsible despite his involuntary intoxication. (E) is incorrect because murder is a specific intent offence (recklessness is not a form of mens rea available).

84
Q

A 16-year-old youth is sentenced to a youth rehabilitation order.

Which of the following cannot form a part of the order imposed?

A 240 hours unpaid work over 24 months.r

B An anger management programme.

C A curfew.

D A residence requirement at a specified address.

E An exclusion from a specified zone.

A

A 240 hours unpaid work over 24 months.

(A) Whilst an order can impose a requirement for 240 hours of unpaid work, it can only be over a maximum of 12 months. All the other options are possible conditions. A youth rehabilitation order can last up to three years and can include a variety of conditions, including supervision (attending appointments with the probation service), unpaid work (completing up to 240 hours of unpaid work over a maximum of 12 months), activities (making restitution to the victim or addressing their offending behaviour), programmes (such as anger management), curfew, exclusion from a specified area, and residence requirements, amongst others

85
Q

A man signs a document in which he names a trustee and declares “I give my 500 shares in X Ltd to my trustee to hold on trust for my children in equal shares”. On the man’s death, a signed transfer of the shares to the trustee is found among his papers, but the title remains registered in the man’s name.

Which of the following statements best describes the status of the trust?

A The trust fails because the share transfer was not delivered to the trustee.

B The trust is enforceable because the trust was declared in signed writing.

C The trust is enforceable because the settlor made every effort to transfer the legal title to the trustee.

D The trust fails because it was not declared in a will.

E The trust is enforceable, and the settlor’s executors hold the shares on trust for his children.

A

The trust fails because the share transfer was not delivered to the trustee.

(A) The trust fails because the share transfer was not delivered to the trustee. The settlor was attempting to create a trust of personalty with a third party as trustee. He needed to declare the trust with certainty, which he has done. His words show that he intended to create a binding obligation, the subject matter is certain (“my 500 shares in X Ltd”), and the objects are the settlor’s children. A declaration of trust of land must be evidenced by some writing, signed by the settlor. In this case the trust property consisted of personalty (the shares in X Ltd) and so signed writing was not essential. However, the settlor also needed to transfer the trust property to the trustee in the appropriate legal manner. In the case of shares, a stock transfer form must be completed and lodged with the company. Legal title passes when the trustee is registered as legal owner of the shares in the company books. The usual rule is that ‘equity will not assist a volunteer’, and if legal title has not passed, the trust will fail. An exception to this rule is the ‘every effort’ test, which applies where the settlor has done everything required of him to transfer the legal title and has put the property beyond his control, but that is not the case here. (B) is incorrect because, as explained above, in addition to making a valid declaration of trust, the settlor needed to transfer the trust property to the trustees. (C) is incorrect because, although the settlor did sign the necessary form, by retaining it in his possession he failed to put the shares outside his control. It was open to the settlor to destroy the transfer. This means that the ‘every effort’ test does not apply. (D) is incorrect because the trust was declared in the settlor’s lifetime and did not need to appear in his will. (E) is incorrect because the trust has failed for the reasons stated above.

86
Q

A man’s neighbour owned an expensive sports car. The man told his neighbour that he wanted to impress his fiancee’s family and asked if he could borrow the sports car and drive it to his fiancee’s parents’ house. The man’s neighbour agreed as long as the man drove only to and from the parents’ house, which the man promised to do. In fact, the man intended to drive the sports car to an abandoned racetrack where he and his friends would speed around in it. While driving the sports car, the man’s friend crashed into a wall, causing serious damage to the car. In fact, the friend had previously damaged three other cars that he had driven around the abandoned racetrack. When the neighbour learned what happened to his car, he contacted the police.

What offence would the prosecution likely charge to secure a conviction?

A Attempted criminal damage.

B Fraud by failure to disclose information.

C Theft.

D Attempted theft.

E Fraud by false representation.

A

Fraud by false representation.

(E) The prosecution would probably charge the man with fraud by false representation. A person commits fraud by false representation if he dishonestly makes a false representation and intends, in doing so, to make a gain for himself or cause a risk of loss to another. Here, the man acted dishonestly when he told his neighbour he was going to drive the sports car to his fiancee’s parents’ house when in fact he was going to drive on a racetrack with his friends, knowing that three previous cars had been damaged by his friend. (A) is incorrect because the facts do not show the man had the intention to commit criminal damage (which is necessary for attempt). (B) is incorrect because fraud by failure to disclose would be difficult to charge, as the prosecution would have to show the man had a legal duty to disclose that he was going to race the car with his friends, and it is not clear any such duty existed. (C) and (D) are incorrect because theft and attempted theft may be difficult to prove because there may not be an intention to permanently deprive, and attempt requires an intention to commit the full underlying offence.

87
Q

An 80-year-old man and his 70-year-old wife made wills last year. The wife’s will leaves all of her assets to her husband. The husband’s will leaves all of his assets to his wife. Last week the couple were killed in a motorway accident. They both died almost instantly, before anyone was able to reach the scene of the accident. Consequently, it was impossible to determine whether the husband or wife died first. The couple married late in life, and they each had children with previous spouses. Both had no other living relatives.

Who will inherit their estates?

A The wife’s children will inherit both estates under the intestacy rules.

B The two estates will be combined and then shared equally between the husband’s children and the wife’s children.

C Both wills fail due to lapse, and the intestacy rules will apply so that the husband’s children inherit his estate, and the wife’s children inherit her estate.

D The husband’s children will inherit both estates under the intestacy rules.

E The husband’s children will inherit the wife’s estate, and the wife’s children will inherit the husband’s estate.

A

A The wife’s children will inherit both estates under the intestacy rules.

(A) The wife’s children will inherit both estates under the intestacy rules. If a beneficiary has predeceased the testator, the gift to them will lapse. In a simultaneous death situation where it is impossible to determine who died first out of a testator and beneficiary, the law of ‘commorientes’ provides that the younger person is taken to survive the elder person for succession purposes. Thus, here the husband will be deemed to have died first because he is older. His assets pass to his wife according to his will. As the husband will be treated as having predeceased his wife, the gift in her will to him lapses. Because that was the only gift in the woman’s will, the will fails, so her estate – which now includes the husband’s estate – passes by intestacy to her children. (B), (D), and (E) are, therefore, incorrect. (C) is incorrect because the husband’s will does not fail due to lapse. The husband is assumed to have died before his wife, and so she is treated as a living beneficiary at the time of his death.

88
Q

A trust of successive interests has been created with a minor life tenant and a remainderman. The income was not used in the first three years of the trust’s creation; rather, it was accumulated.

Which of the following statements about the accumulations is true?

A Accumulations are paid to the life tenant at the age of 18 in all circumstances.

B Accumulations are reinvested as capital to produce further income.

C Accumulations are beneficially funds of the remainderman.

D Accumulations are unavailable during the beneficiary’s minority for their maintenance, education, or benefit.

E Accumulations are paid to the life tenant at age 25 if their interest is vested.

A

B Accumulations are reinvested as capital to produce further income.

(B) Income accumulated during a beneficiary’s minority is reinvested as capital to produce further income. (A) is incorrect because a life tenant is not entitled to accumulations at the age of 18 in all circumstances; it depends on the nature of the interest. (C) is incorrect because accumulations are beneficially the funds of the life tenant and do not become the property of the remainderman. (D) is incorrect because accumulations are available, at the trustee’s discretion, to the minor beneficiary for their maintenance, education, or benefit. (E) is incorrect because a beneficiary who has a vested interest in the income is entitled to the accumulations at age 18.

89
Q

A man is the tenant of a small factory unit on an industrial estate. His business is not doing well and he wants to bring his lease to an end. He met the landlord at the premises last week and handed the keys back to the landlord after clearing the property. The landlord said to him: ‘Don’t worry about the rent. Thank you for the keys’. The landlord has since put a ‘to let’ board up at the premises.

Which of the following best describes the legal effect of the man’s action in handing the keys back?

A The man’s action alone has ended the lease.

B The man’s lease has ended but the landlord can pursue him for any rent arrears due from the date that the keys were handed back to the date that the landlord re-lets the premises.

C The man has forfeited the lease and the landlord has accepted so the lease is ended.

D The man’s actions, when paired with the landlord’s actions, have probably brought the lease to an end by surrender by operation of law.

E The man’s lease has been brought to an end by merger.

A

D The man’s actions, when paired with the landlord’s actions, have probably brought the lease to an end by surrender by operation of law.

(D) The man’s actions, together with the landlord’s actions, have probably brought the lease to an end by surrender by operation of law. Surrenders by operation of law may be effected without writing and may occur, for example, when a landlord and tenant demonstrate by their conduct that there is no longer a continuing lease between them. Here, there was no surrender by deed, but the man and the landlord demonstrated by their conduct that there was no longer a tenancy between them. The man’s handing over of the keys and the landlord’s acceptance of the keys, telling the tenant not to worry about the rent, and putting up the ‘to let’ board is conduct consistent with the end of the tenancy. (A) is incorrect because handing the keys back alone does not mean the man’s lease has ended. The landlord and tenant must together demonstrate by their conduct that there is no longer a continuing lease between them. (B) is incorrect because, as explained above, the tenant and landlord’s actions together must demonstrate the lease has ended (not the tenant’s actions alone). In any event, the landlord’s actions will prevent him from pursuing the tenant for rent arrears since the man’s and the landlord’s actions together demonstrate that the lease has come to an end. (C) is incorrect because forfeiture is a right for the landlord to bring the lease to an end in the event of a tenant’s default, which is not relevant here. (E) is incorrect because merger occurs if the parties agree that the landlord will transfer the reversion to the tenant, which is not the case here.

90
Q

A client instructs a solicitor to assist the client in the purchase of a small business. The client sends the solicitor a cheque for £1,500 to be held on account of costs of the transaction.

Which of the following entries would be the most appropriate entry to record this receipt?

A Credit Cash Account - Client Side; Debit Client ledger - Client Side

B Debit Cash Account - Client Side; Credit Client ledger - Client Side

C Credit Cash Account - Business Side; Debit Client ledger - Client Side

D Credit Cash Account - Business Side; Debit Client ledger - Business Side

E Debit Cash Account - Business Side; Credit Client ledger - Business Side

A

Debit Cash Account - Client Side; Credit Client ledger - Client Side

(B) This receipt is client money so it must be shown as a credit entry on the client side of the client ledger, and the corresponding entry would be a debit on the cash account client side, as it is money that the firm now owes to the client. (A) is incorrect because it shows the entries for a payment from the client account, not a receipt. (C) - (E) are incorrect as this is not business money and, therefore, should not be shown on the business side of the cash account or the client ledger. (C) also is incorrect because in double entry accounting, both entries must be on the same side - business or client.

91
Q

A man is upset that his girlfriend has broken up with him, so he decides to get revenge on her. Intending to cause embarrassment and inconvenience, he goes to her company’s office building and pours oil all over the entrance steps, intending her to slip and fall. The company boss arrives for work first and slips on the oil and falls. This causes the boss, who has a heart condition, to have a heart attack and die.

On these facts, what is the most serious offence the man could be charged with?

A Murder.

B Voluntary manslaughter.

C Involuntary manslaughter.

D Section 20 grievous bodily harm (‘s20 GBH’).

E Criminal damage.

A

C Involuntary manslaughter.

(C) The most serious charge against the man can be involuntary manslaughter. A charge of involuntary manslaughter, specifically unlawful act manslaughter (‘UAM’), is made out here. UAM requires an intentional act, which is unlawful, dangerous, and causes the death of the victim. Here, the man committed an underlying offence of criminal damage, the act was intentional, pouring oil on steps was dangerous, and it caused the death of the boss. The fact that the boss had a heart condition does not break the chain of causation; a defendant must take the victim as they find them. (A) is incorrect as murder requires the intention to kill or cause grievous bodily harm (‘GBH’). The man does not have intention to kill or cause GBH – even against his ex-girlfriend – so even using transferred malice, the charge is not made out. (B) is incorrect as voluntary manslaughter requires the mens rea of murder, which is not satisfied here. (D) and (E) are incorrect as, whilst they might be made out, they are not the most serious offences that could be charged.

92
Q

A woman made a valid will, but she is now thinking of revoking it after an argument with some of the beneficiaries.

Which of the following actions is most likely to fully revoke her will?

A She tears up the front page of the will.She tears up the front page of the will.

B She validly executes a new will which includes an express statement that it revokes all previous wills.

C She telephones her solicitor and asks him to destroy the will, and the solicitor does so whislt they are still on the telephone.

D She writes “cancelled” across the front page of the will.

E She shreds the will by mistake but then is glad that it was destroyed.

A

B She validly executes a new will which includes an express statement that it revokes all previous wills.

(B) The woman can fully revoke her will by validly executing a new will which includes an express statement that it revokes all previous wills. A valid new will which expressly revokes previous wills fully revokes the earlier will. (A) is incorrect. A testator can revoke a will by intentionally destroying it. Destruction of part of the will may result in a partial revocation or a complete revocation if a sufficiently substantial or vital part is destroyed. Whether the destruction of the front page of the will results in a partial or complete revocation depends on what the front page of the will stated. Therefore, this is not the action most likely to fully revoke the woman’s will. (C) is incorrect. When a will is destroyed by someone other than the testator, the testator must be present in order for revocation to occur. (D) is incorrect. The will or page must be fully destroyed in order to be revoked, and simply writing “cancelled” on it is insufficient. (E) is incorrect. The testator must have an intention to revoke the will at the time the will is destroyed. The will was shredded by mistake, so this intention was missing here. It does not matter that the testator was later glad that the will was shredded.

93
Q

A man’s valid will makes a gift of £10,000 to his brother. Five years after making the will, the man crosses through “£10,000” and writes “£15,000” above it. No witness was present at the time that this change was made. The wording relating to the original gift remains legible.

Which of the following statements best describes the effect of this alteration?

A The brother will receive £15,000 provided the alteration was made in ink rather than in pencil.

B The brother will receive £10,000.

C The brother will receive nothing.

D The amendment will invalidate the will, and the intestacy rules will apply.

E The brother will receive £15,000.

A

B The brother will receive £10,000.

(B) The brother will receive the original gift of £10,000. An unattested alteration is a change made to a will without the formalities of the testator’s signature and two witnesses. An unattested alteration made after a will was executed is ineffective. If the original words are apparent, they remain a valid part of the will. Here, the man made an unattested alteration after the will was executed, but the original gift of £10,000 is still legible. Therefore, the original gift remains valid. (A) is incorrect. It does not matter how an unattested alteration is written; it is invalid in both ink and pencil. (C) is incorrect. Because details of the original gift remain visible, the gift of £10,000 remains valid. (D) is incorrect. The alteration will not invalidate the will. (E) is incorrect. The alteration was not properly executed, and so the revised gift does not take effect. If the man had executed the alteration according to the statutory will formalities, the revised gift would be valid.

94
Q

A woman made a will leaving a gift of “£5,000 to each of the children of my brother who attain age 21, whenever born”. The woman has recently died. The woman’s brother has no children currently.

Which of the statements about this gift is correct?

A The gift fails because the brother has no children.

B The gift will fail if the brother dies having had no children who reach age 21.

C All future children of the brother who are born before the brother’s first child reaches the age of 21 will be entitled to £5,000.

D The gift will fail as gifts of this type are uncertain and so can never be valid.

E All future children of the brother who attain the age of 18 will be entitled to £5,000.

A

B The gift will fail if the brother dies having had no children who reach age 21.

(B) The gift will fail if the brother dies having had no children who reach age 21. When a will leaves a gift to each member of a class of beneficiaries who meets a condition, the class closing rules apply to exclude beneficiaries who are not living when the class closes. Generally in this situation, the gift fails when there is no one in the class at the date of the testator’s death. This would result in the gift to the brother’s children failing, as he had no children on the date when the woman died. However, a testator can exclude the class closing rules by using a specific provision to this effect in the will. That is the case here – the will leaves the gift to the children “whenever born”. Therefore, it does not matter that the brother currently has no children, and the gift will extend to any future children of the brother. (A) is incorrect. The gift would fail if the testator had not included the phrase “whenever born”. Instead, the gift is potentially valid, despite the brother currently having no children, as he may have children in the future. (C) is incorrect for the reasons explained above. . The class of beneficiaries will not close when the brother’s first child reaches the age of 21 because of the words “whenever born”. Thus, any child who attains age 21 will receive the gift. (D) is incorrect. Gifts of this type can be valid. (E) is incorrect because the will requires the beneficiaries to reach age 21, not age 18.

95
Q

A settlor transfers property to four trustees to hold on trust for his nieces and nephews. The trust deed contains no express powers dealing with the appointment of trustees. One of the trustees wishes to retire.

Which of the following best describes the legal position regarding the trustee’s power to retire?

A The trustee may retire but must be replaced.

B The trustee may retire without replacement.

C The trustee may retire without replacement provided the settlor consents in writing.

D The trustee may retire without replacement provided his co-trustees consent.

E The trustee may retire without replacement provided his co-trustees consent by deed.

A

E The trustee may retire without replacement provided his co-trustees consent by deed.

(E) A trustee may retire without replacement provided: (1) he obtains the consent by deed of all his co-trustees and the person, if any, given power to appoint new trustees by the trust instrument, and (2) he leaves in office at least two trustees or a trust corporation. The facts state that the trust deed contains no powers dealing with the appointment of trustees, so the trustee requires only the consent of his co-trustees. (B) and (D) are incorrect because the co-trustees must consent by deed. (C) is incorrect because the settlor made no express provision in the trust deed reserving power to appoint new trustees. (A) is incorrect because three trustees will remain in office after the fourth trustee’s retirement, which meets the requirement that a retiring trustee leave in office at least two trustees

96
Q

A married couple were involved in a serious car accident three weeks ago. The husband died in hospital a few hours after the accident. The wife died of her injuries two weeks later. Neither had a will, and they had no children. The man’s father, brother, and his brother’s son are still alive. The wife’s sister is also alive.

Who is entitled to share in the distribution of the man’s estate?

A The man’s brother only.

B The man’s father and brother.

C The man’s father, brother, and the brother’s son.

D The man’s father only.

E The wife’s sister only.

A

(D) The man’s father only is entitled to the man’s estate. The rules of intestate succession apply when a person dies without a valid will. Under these rules, where there is a spouse or civil partner but no issue, the spouse or civil partner will inherit the entire estate, provided they survive the deceased by 28 days.

Here, the man’s wife died within 28 days of the man, and so the man’s estate will not pass to her or her sister. Instead, applying the intestacy rules, the man’s father will inherit the whole estate because parents of the deceased are entitled to the estate before siblings. Accordingly, (A), (B), (C), and (E) are incorrect.

97
Q

An executor is administering the estate of a man who died recently. The man ran his own business as a sole trader. The man’s assets (all of which pass to his son) total £22,000. In addition to funeral expenses (£3,000), the man owed debts to the following creditors:

£2,000 to the one employee of his business for the past two months’ wages;

£20,000 to trade suppliers; and

£5,000 to his wife.

Which of the following is true with respect to payment of the man’s debts and funeral expenses?

A All of the creditors will receive an equal proportion of the sums owed to them.

B The man’s wife will receive full payment of the £5,000 owed to her.

C The trade suppliers will receive full payment of the sums owed to them.

D The funeral expenses and the employee will be paid in full, but the trade suppliers and the wife will receive only an equal proportion of the sums owed to them.

E Only the funeral expenses will be paid in full.

A

(E) Only the funeral expenses will be paid in full.

This is an insolvent estate, as there are insufficient assets to pay all expenses and debts in full. All debts are unsecured and must be paid in the prescribed order. The order is funeral and administration expenses, preferred debts, ordinary debts, interest on preferred and ordinary debts, and deferred debts. Each creditor ranks equally in a category, and their payments abate proportionally. Here, the funeral expenses must be paid first, and there are sufficient funds to pay the £3,000 in full. Next are preferred debts, which are the wages and salaries of the deceased’s employees in the four months prior to death, up to a maximum of £800 each. Therefore, the man’s employee will receive only £800 as a preferred debt payment. Next are ordinary debts, comprising the trade suppliers and the balance of the sum owing to the employee. At this point, there is £18,200 left to pay £21,200 of ordinary debts. As there are insufficient funds to cover the sum due to the ordinary creditors, they will receive the same proportion of the sum owed to them. Finally, loans from the deceased’s spouse are deferred debts, so the man’s wife is a deferred creditor who has the lowest priority. She will receive nothing, as there are no funds left to pay her. (A) is incorrect. Debts are paid according to an order of priority. It is only within the priority categories that creditors receive equal proportions of the sums owed to them. (B) is incorrect. The wife is a deferred creditor and will receive nothing here. (C) is incorrect. There are insufficient assets to pay the trade suppliers in full. (D) is incorrect. Whilst the funeral expenses are paid in full, the employee will not be fully paid, as explained above. The trade suppliers will receive a proportion of the debts owed to them, but the wife will receive nothing as a deferred creditor.

98
Q

An owner of a restaurant rents their premises under the terms of a lease which is protected by the Landlord and Tenant Act 1954 (Part II). The lease is due to end in 11 months. The restaurant owner wants to remain in the premises at the end of the term under new lease terms and instructs a solicitor to send the landlord a section 26 notice to that effect. The solicitor drafts and sends the notice the next day.

How long does the landlord have to respond to the notice?

A 30 days.

B 30 working days.

C Two months.

D Five months.

E Nine months.

A

C Two months.

(C) Under the Landlord and Tenant Act 1954 (Part II), a tenant may serve a section 26 notice on the landlord requesting a new lease at the end of their lease term. The notice must be served within six to 12 months before the end of the lease term. The landlord then has two months to inform the tenant whether they are willing to enter into a new lease or to raise a statutory ground to terminate the tenancy. (A) and (B) clearly indicate the wrong timescale. (D) is incorrect because it is based on the latest date (11 months minus five months equals six months) at which a landlord could serve a section 25 notice declaring an intent to terminate the tenancy. But after the tenant serves a section 26 notice, the section 25 notice period is irrelevant. (E) is incorrect because it is based on two months from the end of the lease term rather than two months from the time of the notice.

99
Q

A car owner asked a mechanic to change the tyres on his car. They agreed to a price of £200 to be paid when the car was picked up. The mechanic changed the tyres and telephoned the owner to come pick up the car and pay for the tyres. Not wanting to pay for the tyres, the car owner told his friend that the car was ready, but that he could not go get it. He explained the tyres were already paid for, gave the friend a key, and asked the friend to pick up the car. The friend duly did so.

Will the car owner likely be found guilty of theft?

A No, because it was the car owner’s own car.

B Yes, because the car owner had promised to pay the mechanic for his work when he came to get the car.

C No, because the car owner only really stole the tyres.

D Yes, because the friend took the car without the mechanic’s permission.

E No, because the friend took the car and not the owner.

A

D Yes, because the friend took the car without the mechanic’s permission.

(D) The car owner would likely be found guilty of theft. Theft is the appropriation of property belonging to another with the intention to deprive that person of it permanently. It is possible to steal your own property if someone – for example, a mechanic – has superior right to possession at that time. Therefore (A) is wrong. (B) is wrong because even if the car owner had not made a promise to pay for the tyres, he incurred the obligation by having the work done. (C) is wrong because the mechanic has superior title to the car at that time, not just the tyres. (E) is also wrong because a person can use an innocent agent, in this case the car owner’s friend, to commit a crime.

100
Q

A defendant is charged with battery, a summary only offence. He appears in the Magistrates’ Court for his first court appearance.

What is the Magistrates’ Court likely to do next?

A Decide whether the Magistrates’ Court’s sentencing powers are adequate.

B Send the matter directly to the Crown Court.

C Explain to the defendant that he may indicate whether he would plead guilty if the matter were to proceed to trial.

D Tell the defendant he must choose between Magistrates’ Court and Crown Court.

E Ask the defendant for his plea.

A

E Ask the defendant for his plea.

(E) The Magistrates’ Court will ask the defendant for his plea. Summary only offences can be dealt with by the Magistrates’ Court only. At the first hearing, the court will ask the defendant for his plea. If the defendant pleads guilty, the court will proceed to sentence unless pre-sentence reports are required. If the defendant pleads not guilty, a trial date will be set. (A), (C), and (D) are incorrect because they are parts of the plea before venue process, which applies to either way offences. (B) is incorrect because indictable only offences are sent directly to the Crown Court.

101
Q

An employee does some overtime at work. They receive payment for the overtime in their next pay cheque. The following month, they are paid for the same overtime again. Having heard that redundancies are coming, the employee decides to keep the overpayment as a buffer even though they know they are not entitled to it.

Is the employee guilty of theft?

A No, as there is no appropriation.

B Yes, as the employee knows they are being dishonest.

C Yes, as the employee is being dishonest by the standards of reasonable, honest people.

D No, as there is no dishonesty.

E No, as it is a clerical error by the employer.

A

Yes, as the employee is being dishonest by the standards of reasonable, honest people.

(C) The employee is guilty of theft. Theft is the dishonest appropriation of property belonging to another with the intention of permanently depriving the other of it. By keeping the overpayment, the employee has effected an appropriation. The test for dishonesty is that of the reasonable, honest person; here the employee falls short of that standard. For those reasons, (A), (D), and (E) are incorrect. (B) is incorrect as it sets the test for dishonesty as subjective, whereas it is objective. The employee does not have to know that they are being dishonest; they have to be dishonest by the standards of a reasonable, honest person.

102
Q

A woman has owned a field for 30 years. For the entire period of her ownership she has walked over a strip of land belonging to her neighbour to gain access to the public highway. She has never asked her neighbour for permission to go across their land nor has the neighbour ever spoken to the woman about it.

Which of the following best describes the woman’s rights to the strip of land?

A She has a licence to use the strip of land.

B She has established an easement by prescription.

C She has no rights and should ask the neighbour to formalise the situation via a deed of easement.

D She has an easement implied by existing use.

E She has no right to use the strip of land and cannot obtain any such rights at this point.

A

She has established an easement by prescription.

(B) The woman has established an easement by prescription. An easement by prescription arises by long use, that is, a party must show that they have (1) used the benefit unchallenged for over 20 years, and (2) used the benefit as of right, meaning they did not ask for permission or make any payment for the use of the benefit. Here, the woman has used the strip of land for over 20 years unchallenged and has not asked permission to do so, thus she is likely to have acquired a right to use the strip of land by prescription. (A) is incorrect because a licence is a personal right to use land in some way and entails the landowner’s permission. (C) is incorrect because, as explained above, the woman has established prescriptive rights to the strip of land. Further, by asking the neighbour to formalise the situation, she would defeat any claim for a prescriptive right that she may have because the use claimed must be ‘as of right’. In other words, if permission is sought, this is a tacit acceptance of the prior right of the neighbouring landowner. (D) is incorrect because an easement implied by existing use applies on a sale of part of a land, which is not relevant here. (E) is incorrect because, as stated above, the woman has acquired a right to use the strip of land by prescription.

103
Q

An elderly but competent testator is in hospital. He gives his nephew a key and instructs the nephew to go to the testator’s home, get the safe out from under the testator’s bed, open it with the key, and tear the will up that is inside the safe. The nephew does what he was asked, and the testator dies a few days later.

Was the testator’s will validly revoked?

A Yes, because the testator was competent.

B Yes, because the testator had the intent to revoke his will and may do so through the actions of a third party.

C No, because nothing indicates the testator had the intent to revoke at the time the nephew tore up the will.

D No, because the testator was not present when the will was destroyed.

E No, because the testator did not physically destroy the will himself.

A

D No, because the testator was not present when the will was destroyed.

(D) The testator’s will was not revoked because he was not present when the will was destroyed. A will may be revoked by intentional destruction, and the destruction may be by a third party acting on behalf of the testator and at the testator’s direction. However, the testator must be present at the time of the destruction. Since the testator was not present when his nephew tore up his will, the will was not validly revoked. (A) and (B) are therefore incorrect. (C) is incorrect because it appears the testator intended the will to be revoked; the problem with the revocation is that he was not present. (E) is incorrect because a will may be revoked by destruction by a person in the testator’s presence and acting at his direction.

104
Q

A trustee manages a trust on behalf of a beneficiary. In breach of trust, the trustee removed £10,000 from the trust bank account and placed it into his personal current account. At the time the £10,000 was deposited in his personal current account, the account was overdrawn by £2,000. The trustee is now bankrupt.

How can the trust beneficiaries recover the £10,000 that was taken by the trustee?

A The beneficiaries can claim a charge against the estate of the trustee to recover the £10,000 in full.

B The beneficiaries can claim a charge over the account for the £10,000 in full, and the bank would have a personal action against the trustee on the £2,000.

C The beneficiaries have a personal action against the trustee to recover the £10,000, together with a claim for an amount of interest.

D The beneficiaries can claim a charge over the balance of the account, but they must make a personal claim against the trustee to recover the £2,000.

E The beneficiaries can claim a charge over the balance of the account, and the £2,000 can be recovered from the bank in a personal action as the bank is a recipient of trust property.

A

D The beneficiaries can claim a charge over the balance of the account, but they must make a personal claim against the trustee to recover the £2,000.

(D) The beneficiaries can recover £8,000 by claiming a charge over the balance of the account. The £2,000 used to discharge the overdraft is regarded in law as dissipated, so the beneficiaries must make a personal claim to recover it. (A) and (B) are incorrect because the beneficiaries can claim a charge over the account for only £8,000. (C) is incorrect because the beneficiaries have a proprietary claim over the £8,000 remaining in the bank account. (E) is incorrect because a recipient of trust property can be held personally liable for receipt of the property only if they knowingly received the property in breach of trust, and here nothing indicates the bank knew about the breach.

105
Q

A trustee is appointed under the terms of a trust of concurrent interests with three beneficiaries. The trust property consists of 10,000 shares in a private limited company. Under the terms of the trust, the trustees may use the shareholding to appoint themselves or others as directors of the private limited company. In exercise of this power, the trustees appointed one of their number to the board of directors of the private limited company. The appointed trustee received remuneration as a director of £38,000 per annum.

Is it permissible for the trustee to retain the remuneration?

A No, unless the trustee provided their services as a professional trustee.

B No, unless the trustee provided full and frank disclosure to the beneficiaries.

C No, even though the trustee has acted lawfully and in the proper discharge of their functions as trustee.

D Yes, if the trustee has a reasonable expectation of receipt of the remuneration in the terms of the trust.

E Yes, if the trustee has made an immediate declaration of a conflict of interest to the beneficiaries of the trust.

A

C No, even though the trustee has acted lawfully and in the proper discharge of their functions as trustee.

(C) It is not permissible for the trustee to retain the remuneration even though the trustee has acted lawfully. A trustee who obtains remunerative employment by virtue of the trusteeship holds the remuneration on constructive trust for the trust beneficiaries. This principle has been applied most commonly to cases like the one here where a trustee has been appointed to be a company director by virtue of the fact that they hold shares in the company as trustee. The trustee, therefore, cannot keep the remuneration. (A) is incorrect because it is irrelevant whether the trustee provided their services as a professional trustee. (B) and (E) are incorrect because the trustee may not retain the remuneration even with the provision of disclosure or a declaration of conflict of interest to the beneficiaries. (D) is incorrect because the trustee may not retain the remuneration even with a reasonable expectation of receipt of remuneration in the terms of the trust.

106
Q

A settlor transfers property to two trustees to hold on trust for his children, all under the age of 18. One of the trustees is a solicitor. The trust deed contains no express powers dealing with trustees’ powers to charge for their services.

Which of the following best describes the solicitor’s power to charge his normal professional charges for the time he spends on trust matters?

A He may charge providing the settlor consents in writing.

B He may not charge because there is no express power in the trust deed.

C He may charge providing his co-trustee consents in writing.

D He may not charge because the beneficiaries are under 18.

E He may charge without his co-trustee’s or beneficiaries’ consent.

A

C He may charge providing his co-trustee consents in writing.

(C) The solicitor may charge for his services providing his co-trustee consents in writing. The normal rule is that trustees may not profit from their trusteeship, so a trustee may not charge unless there is an express power in the trust instrument. However, by statute, a professional trustee may charge their normal professional charges for their services in relation to the trust provided that they are not a sole trustee and that their co-trustees consent in writing to their charges. (A) is incorrect because once the trust has come into existence the settlor retains no further control unless there are express provisions in the trust instrument. (B) is incorrect because the statutory power applies. (D) is incorrect because there is no requirement for the beneficiaries to consent under the statutory power. (E) is incorrect because a professional trustee may only charge with their co-trustees’ written consent

107
Q

Two people enter into a contract for the sale of a piece of land. The buyer wants to develop the land because it is in a beautiful location. The buyer also believes that they will make a lot of money when the land is developed and sold on. The seller did not complete the transaction on the agreed completion date.

The buyer was very upset and put a number of adverts in the local newspaper naming the seller and making detrimental comments. The buyer also told the seller that if the seller did not reduce the sale price, the buyer would ruin the seller’s reputation. The buyer then served a notice to complete on the seller. The seller neither lowered the price nor completed the conveyance by the time in the notice. The buyer has now made an application to court for an order for specific performance.

Is the buyer’s application likely to be successful?

A Yes, because specific performance is a remedy which is available to the buyer as of right.

B Yes, because specific performance is available in conveyances of property because land is considered unique.

C Yes, because the seller is in breach of contract having failed to complete on the contractually agreed date.

D No, because specific performance is an equitable remedy and the buyer has not acted fairly.

E No, because the only remedy available before completion is an action for damages.

A

D No, because specific performance is an equitable remedy and the buyer has not acted fairly.

(D) The buyer’s application for specific performance is not likely to be granted because the buyer has not acted fairly. Specific performance is an equitable remedy and ‘he who comes to equity must come with clean hands’. If the applicant has not acted properly with respect to the subject matter of the action, a court is very unlikely to grant the application. (A) is incorrect because specific performance is an equitable remedy and is not available as of right – it is a matter of discretion. (B) is incorrect because whilst specific performance is available for property transactions because property is considered unique (it is not available when money damages are a viable remedy), this choice fails to address the other requirement – that the applicant must have clean hands. (C) is incorrect because, again, it fails to take into account the need for clean hands. (E) is incorrect because it is not true; specific performance and rescission are other possible pre-completion remedies.

108
Q

A defendant is facing trial for fraud by false representation. It is alleged that the defendant made loan applications in the name of his business and used the funds for his own benefit. The prosecution wish to adduce the following hearsay evidence:

  • Company account documents for the defendant’s business.
  • The written witness statement of an employee of the defendant’s business who has moved to Australia and is unable to attend for trial.
  • Oral evidence from the defendant’s friend who states that the defendant admitted to him in the pub that he was using company loans to pay for his motorbike hobby.

Which of these hearsay statements will be admissible?

A All of them.All of them.

B The account documents and employee’s statement only.

C The account documents and friend’s oral evidence only.

D The employee’s statement and friend’s oral evidence only.

E None of them.

A

All of them.

(A) All of the hearsay statements will be admissible. They are all hearsay, as they are all statements (whether oral or written) made outside of court adduced to prove the truth of their content. Hearsay is not admissible unless it falls under one of the exceptions to the rule against hearsay. All three of these statements fall into a hearsay exception. Business documents are admissible under statute. Statute also preserves evidence when the witness is not available and it is not reasonably practicable to secure their attendance. Confessions are admissible under rule of law. Therefore, (B), (C), (D), and (E) are incorrect.

109
Q

A defendant is standing trial for actual bodily harm (‘ABH’) in the Magistrates’ Court presided over by a District Judge. At the start of the trial, the prosecution wish to make an application to adduce evidence of a confession. The prosecution allege the defendant admitted committing the offence to a friend in the pub the night before the start of the trial, and the prosecution wish to adduce this evidence as proof of his guilt.

How will the court determine whether evidence of the confession can be admitted?

A The District Judge will hold a voir dire in absence of the jury to determine the admissibility of the evidence.

B The District Judge will determine the admissibility in presence of the jury.

C The District Judge will hear the application to adduce the evidence and then recuse himself if the evidence is inadmissible.

D The District Judge will hear the application to adduce the evidence and continue to act even if the evidence is held to be inadmissible.

E The District Judge will enlist the assistance of another judge or panel of magistrates to hear the application.

A

D The District Judge will hear the application to adduce the evidence and continue to act even if the evidence is held to be inadmissible.

(D) The District Judge will hear the application to adduce the evidence and will continue to act even if the evidence is held to be inadmissible. In the Magistrates’ Court, the bench decide matters of both fact and law. If the bench rule a confession inadmissible, they must then put the confession from their mind and decide the case without it. (A) and (B) are incorrect, as there is no jury in the Magistrates’ Court. (C) is incorrect, as the District Judge will continue to act if the evidence is deemed inadmissible. (E) is incorrect, as the District Judge will hear the application himself.

110
Q

A paralegal in the post-completion department of a busy conveyancing firm has checked their case management system and notes that the official search with priority for a property with a registered title was submitted on behalf of a client 20 working days ago.

Within what timeframe must the paralegal make the application for registration?

A 11 calendar days.

B It doesn’t matter, the priority period has already been missed.

C 30 calendar days.

D 30 working days.

E 11 working days

A

E 11 working days

(E) The priority period ends on the 30th working day after the day on which the official search application was submitted. Thus, since the official search was submitted 20 working days ago, the paralegal has a further 11 working days within which to make the application to His Majesty’s Land Registry. Thus, the remaining answers are incorrect.

111
Q

A firm charges a client £4,000 plus VAT of £800 in profit costs for work carried out in relation to a conveyance. The client complained that the costs exceeded the estimate that the client was given when he instructed the firm. In an attempt to keep the client happy, the firm decides to reduce the bill to £3,500 with VAT reduced to £700.

Which of the following entries would the firm make to record the appropriate abatement of fees?

A Debit profit costs and HMRC ledgers - Credit Client ledger (business side)

B Debit profit costs and HMRC ledgers - Debit Client ledger (business side)

C Credit profit costs and HMRC ledgers - Credit cash ledger (business side)

D Debit Client ledger (business side) - Debit Client ledger (client side)

E Credit profit costs and HMRC ledgers - Debit Client ledger (business side)

A

Debit profit costs and HMRC ledgers - Credit Client ledger (business side)

(A) is the only correct combination of entries, all of the others are incorrect. The firm has already delivered its bill; therefore, where the firm has agreed to abate this bill, the entries in the profit costs ledger, the HMRC (VAT) ledger, and the client ledger must be reduced by the amount of the reduction. This is shown as a credit (CR) entry on the business side of the client ledger of £500 relating to the profit costs and £100 for the VAT. In addition to this, the corresponding profit costs and VAT ledgers must also be reduced. This is shown as a debit (DR) entry of £500 on the profit costs ledger and £100 on the HMRC (VAT) ledgers.

112
Q

A trustee pays £10,000 of trust money into his own bank account, which has an existing balance of £5,000. He buys shares for £5,000 and spends the remaining £10,000 on living expenses. Last week the trustee paid his salary of £4,000 into the bank account. The breach has now been discovered, and the trustee’s creditors are bringing bankruptcy proceedings. The shares are now worth £6,000.

Which assets can be claimed for the trust in a proprietary claim?

A None.

B The shares.

C Only a charge over the shares for £5,000.

D A charge over the shares for £5,000 and £4,000 from the bank account.

E The shares and £4,000 from the bank account.

A

B The shares.

(B) The trust can claim the shares in a proprietary claim. Where a trustee mixes trust funds with his own in a bank account, the trust may claim a charge over the account for the amount of trust money in it. When funds are withdrawn from the account, the trustee is treated as spending his own money first. However, where the balance of the fund has been dissipated, the beneficiaries may claim any asset purchased from the account before the balance was dissipated, or a charge over any such asset. If all the trust funds have been drawn out of the trustee’s bank account, the trust has no claim over money belonging to the trustee which is subsequently paid into the account. Following these rules, the initial presumption was that the shares were purchased with the trustee’s own £5,000. However, since the remaining trust money was spent on living expenses, the trust may claim the shares, or a charge over the shares for £5,000. Since the shares have increased in value, the trust will adopt the shares as representing £5,000 of trust money. The trust has no claim against the money in the bank account. (A) is incorrect because the trust may claim the shares, even though the usual rule is that the trustee is treated as spending his own money first. (C) is incorrect because the trust will take the option of claiming the shares because they have increased in value. (D) and (E) are incorrect because the trust cannot claim the £4,000 paid into the account after all the trust funds were withdrawn.

113
Q

A property owner has a house with a large garden. An estate agent has told the property owner that the garden would be big enough for another house to be built. The property owner would like to build a second home and divide the property but presently does not have enough money to undertake the work. She telephones her solicitor for advice. The solicitor advises that they could obtain outline permission from the local authority so that the work can be commenced when the property owner obtains sufficient funds.

The property owner instructed the solicitor to obtain planning permission. The solicitor presented the property owner’s plans to the local authority, which granted outline permission for the project, reserving the matter of the materials that may be used to build the second home.

Which of the following best describes the explanation of the results that the solicitor will give the client?

A The owner may start the development at any time but must further apply to the local authority for approval of the size of the material that will be used to build the house.

B The owner may start the development at any time provided she reserves the material needed to build the house before construction begins.

C The owner must seek further approval from the local authority within three years as to the materials to be used to build the second home and must begin construction within two years from approval.

D The owner must seek further approval from the local authority within two years as to the materials to be used to build the second home and must begin construction within three years from approval.

E The owner must seek further approval from the local authority within three years as to the materials to be used to build the second home and must begin construction within three years from approval.

A

C The owner must seek further approval from the local authority within three years as to the materials to be used to build the second home and must begin construction within two years from approval.

(C) The solicitor should advise the owner that she must seek further approval from the local authority within three years as to the materials to be used to build the second home and must begin construction within two years from approval. Outline permission is permission from a local authority which broadly approves a property owner’s development plans subject to further approval of any reserved matter. When outline permission is given the property owner must seek approval of the reserved matters within three years of the grant of outline permission and must start the work within two years from approval of the reserved matters. The other choices are incorrect because they provide the wrong timescales. (B) is also incorrect because it fails to properly describe what a reserved matter is.

114
Q

A solicitor is acting for the buyer of a property with an unregistered title. The solicitor has carried out a pre-completion search, namely a full land charges search, against the full name of the seller.

What priority period is conferred by the full land charges search result?

A 30 working days.

B 14 working days.

C 15 working days.

D Two months.

E There is no priority period conferred by this search, as the title is unregistered.

A

C 15 working days.

(C) If title is unregistered, a full land charges search against the full name of the seller confers a priority period of 15 working days; that is, the buyer will be protected against (have priority over) applications for land charges against the owner’s name made during the 15-day period if the transaction completes within the 15-day period. (A) is incorrect as 30 working days is the priority period during which the purchase must be completed and registered following an official search with priority, which is carried out in a registered title transaction. (B) is incorrect as 14 days is the deadline for submission of the Stamp Duty Land Tax return following completion of a purchase. (D) is incorrect, as two months is the time frame within which an application for first registration application must be made to His Majesty’s Land Registry following the purchase of an unregistered title. (E) is incorrect because, as explained above, the full land charges search result confers a 15 working day priority period.

115
Q

A woman died on 20 December 2022, leaving her estate to her son. The woman’s son is independently wealthy and, as such, wishes the assets to pass directly to his children.

By which date must the woman’s son make a variation of the will if it is to be effective for both inheritance tax and capital gains tax?

A 4 May 2024

B 20 December 2024

C 31 December 2024

D 30 June 2025

E 30 June 2023

A

B 20 December 2024

(B) 20 December 2024. The variation must be made within two years of the deceased’s death

116
Q

A man and a woman enter into a valid deed of easement. The deed of easement grants a right-of-way for the woman over the man’s land for the benefit of her adjoining land. Title to both pieces of land is unregistered. The man has expressed interest in selling his land and the woman is worried that he might fail to tell a buyer about the woman’s right of way.

What can the woman do to protect her position?

A Register an equitable easement land charge against the man’s name.

B Register an estate contract land charge against the man’s name.

C Register a notice over the man’s title.

D Register a caution against dealings over the man’s title.

E Register a caution against first registration over the man’s title.

A

E Register a caution against first registration over the man’s title.

(E) The woman should protect her position by registering a caution against first registration over the man’s title. An owner of an interest in unregistered land can register a caution against first registration, which gives an early warning of registration to the cautioner, allowing them to protect their interest when an application for first registration of the land is submitted. Here, the sale of the man’s unregistered land will trigger an application for first registration. Thus, the woman should register a caution against first registration, which will ensure she is notified when the land is about to be registered so that she can protect her interest in the land (that is, her right-of-way). (A) is incorrect because the woman’s right of way is not an equitable easement. The parties entered into a deed of easement, which means she has a legal easement. (B) is incorrect because the facts do not indicate the presence of any estate contract, and this form of protection is therefore irrelevant. (C) is incorrect because notices are relevant to the protection of interests in the registered system; here, the land is unregistered. (D) is incorrect because there is no such form of protection as a caution against dealings. As explained above, the woman should register a caution against first registration.

117
Q

A woman died last month, owning a house, a Toyota car, and £50,000 in a bank account.

Her will, made 10 years ago, included the following provisions:

(1) I give my Audi car, registration number VE17 XKD, to my cousin.

(2) I give the remainder of my estate to my daughter.

There are no other relevant clauses.

What will be the result of the gift of the Audi car under clause (1) of the will?

A The cousin will inherit the woman’s Toyota car.

B The gift of the Audi car will adeem.

C The gift of the Audi car will lapse.

D The woman’s executor must purchase an Audi car for the cousin if there are sufficient funds in the estate.

E The cousin will receive a sum of money instead of the Audi car, and the daughter will inherit the Toyota.

A

B The gift of the Audi car will adeem.

(B) The gift of the Audi will adeem. The gift of the Audi to the cousin is a specific legacy – a gift of a specified part of the estate that is clearly identified at the time of the will’s execution. Under the doctrine of ademption, a specific gift will fail if it is no longer part of the testator’s estate, is subject to a binding contract for sale, or no longer meets the description in the will. As the woman no longer owned the specific Audi at the date of her death, the specific legacy will fail, and the cousin will receive nothing. (A) is therefore incorrect. (C) is incorrect. Lapse of a gift arises when a beneficiary dies before the testator, which has not occurred here. (D) is incorrect because it states the rule for general legacies. If the subject of a general legacy is not in the estate, the beneficiary is entitled to require the executor to purchase it if the estate has sufficient funds. (E) is incorrect because the cousin is not entitled to a sum of money instead of the Audi car.